You are on page 1of 122






‫‪500 new Q‬‬









































































‫❤‬
‫إذكرونا و والدينا في دعائكم



‫في حال وجود أي خطأ

‫يرجى تزويدنا بالحل الصحيح مع ا?صدر

‫على قناة التليقرام




‪https://t.me/obgynq‬‬


Full-term women came in labour she’s 5cm dilated , 60% effaced, station -3, after 6 hrs of regular
contractions she’s 7 cm dilated, station -1 on vaginal exam there was retraction ring , what’s most
likely dx?

A. Macrosmia

B. Obstructed labour ✅

C. Preterm labour

Pt has infertility dyspareunia and dysmenorrhea, what is the best way to check tube patncy >

Hysteroscopy,

hysterosonography,

laparoscopy with die, ✅

Pelvic U/S

14- lady mised period which most appropriate time of us

1- 11-13 weeks✅

2- 16 - 18 weeks

3- 18 - 22 weeks

Preganat 38 weeks with typical preeclampsia with HELP syndrome

Prepphral blood smear show halment cells

- Plasma exchange

- urgent delivery ✅

40y/o female patient underwent PAP smear histopathology showed ASCUS, your next step?

A. Do HPV test✅

B. Colposcopy.

C. Re-evaluate after 6 months

D. Surgery

Dusky changes on cervix + obese + 48 + LmP 2 months‫


؟‬
Bhcg ✅

Pelvic us

Homan test

Pregnant lady came for check up at 20 weeks gestation High hba1c

baby at risk of?

Congenital malformations✅

Chromosomal abnormality

female last pregnancy 15 years ago, amenorrhea for 7 months, negative pregnancy test, wishes to
get pregnant, what is the most appropriate investigation?

A. FSH, LH✅

B. Hysterosalpingogram

C. Endometrial sampling

Merrell
Submucosal fibroid removal in a female wants to preserve
her fertility

A) hystroscopic resection by myomectomy ✅

B) Lap myomectomy

C) Lap hystrectomy

24 years female diagnose with endometriosis she took NSAID but no benefits. What is Most
appointment mx?

A. laparoscopic excision and ablation✅ “ if no OCP”

B. hysterectomy

toucoltic SE:

A-Oliguria

B-Palptation ✅

C-Abdominal pain

D- vaginal bleeding

^terbutaline

Gravid women came with PROM you administered a beta-memetic tocolytics drug what is the side
effect :

A- abdominal pain

B- palpitation✅

C- diarrhea

D- bad odor

pregnant lady has an epilepsy and smoke then she got premature labor asking about

what was the big risk factor

A-smoking✅

B-epilepsy

The baby heart auscultation on the mother umbilicus ,,which is the presentation?

-sholder ✅

-face

-breech

Female 31w present to ER with labor pain, 4 w ago she was diagnosed with PPROM, now the liquor [
or somthing like that ] is exposed, and she has 3 contractions in 10 min. What do you want to do?

A- High vaginal swap for culture and sensitivity ✅

B- IOL

C- augmintation of labor

D- Fetal blood sampling for ph and liquor

Endometriosis , took nsaids , no improve tt

A. Laparoscopy

B. OCP✅

Pregnant Female GA 37 weeks with bicornate uterus, fetal kick appreciated mainly in lower
abdomen, leopold manuver showed global soft presenting part with sluggish ballottment by first and
second pelvic grip, fetal heart sound detected at the level of maternal umbilicu, what is the
recommended way to deliver ?

A- spontaneous vaginal

B- foreceps

C - ventose

D- cesarean sectoion✅

Pregnant presented in 2nd trimester with signs of anemia + had severe vomiting in 1st trimester.
Labs showed mcv 112, hgb 9. Dx:

A- physiological anemia

B- folate deficiency✅

C- b12 deficiency

D- iron deficiency

90 years old comorbid (i don’t remember exactly maybe DM and something else ) known pelvic
organ prolapse grade 3 what is your managment at this level

Pessary✅

Hysterctomy

Idon’t remember the remaining options

33 years old women with heavy menstraution. During examination noticed

firm fundal mass (Next in mangment )

A-Ct

B-us ✅

C-mri

D-biopsy

Pregnant known of sickle cell anemia, you want to give her folic acid,

what is the right prescription for her?

A- 5 mg folic acid till 12 week

B- 5 mg folic acid till she gave birth ✅

C- 5 μg folic acid till 12 week folic acid till 12 week

D- 5 μg folic acid till 12 week folic acid till she gave birth

Yellow green discoloration and husband has urethral discharge, management?

A. fluconazole

B. other antifungal

C. metronidazole✅

Pregnant with history of uterine fibroid, 34 gestational age (not sure) presented with abdominal pain
and fever. Examination unremarkable with closed cervix no passage of tissue.

Which of the following is the most appropriate management?

A. Observation ✅

B. Cesarean section

C. Induction of labor

In 30s and has fibroid 10cm

With heavy bleeding and doctor want to do hysterectomy but her refuse .. what the defentive
management:

A-Artery embolismation ✅

B-OCP

C-Observer

Breastfeeding women ask about contraception:

A. Progesterone only

B. IUD ✅

C. Compound drug

I got a q today about pregnant with hx of herpes simplex and RHD with Ms

which of options indicate using of forceps

Hx of hopes simplex

MS✅

Multipara patient known to have Asthma, and something else, and has a history of taking ABx for
rheumatic fever and have mitral stenosis. Which of the following from her history indicate the use of
forceps? "Exactly written like that".

A-Multipara

B-Asthma

C-Mitral stenosis ✅

D- Condition that I didn't remember

20 something multiparous in the 2nd stage of labor History of herpes and asthma and mitral stenosis
Which of the following indicate the use of forceps? ICQ

- mitral stenosis✅

- prolonged labor

- i dont remember the rest

47 y/o women complaining of AUB for 6 months. She wants to take OCP. What is the most important
test to do before treatment?

A- FSH

B- TSH

C- Pelvic US

D- Endometrial biopsy✅

Which of the following is not contraindicated for vaginal delivery:

A- transverse lie

B- classical CS

C- occipito posterior✅

Patient has heavy bleeding and usually has to change tampon 15-30 times/day, recently she has
been engaged in intercourse with multiple partners to conceive, on examination: retroverted uterus,
decreased uterine mobility, and tender uterosacral ligament with nodularity. What’s the diagnosis?

A. Endometriosis. ✅

B. PID

C. Fibroid

D. Adenomyosis

GBS vaginal swab screening in pregnant women

a. 15 weeks

b. 25 weeks

c. ✅ 35 weeks

d. 40 weeks

Pregnant lady not having chicken box before and antibody is negative ask about with is next
important step?

A- give varccila vaccine

B- Give IG

C- Give prophylaxis

D- Avoid exposure ✅

38 weeks IUGR vertex what type of delivery?

A- Ventos

B- Forceps

C- C/S

D- Spontaneous normal vaginal delivery ✅

Pregnant 26w , uti tt

A. Nitrofurantoin ✅

Pregnant forgot LMP , what the accurate way to detect gestation age:

A. Femur length

B. Head circumference ✅

Note: (no crown-rump length im choices)

Boys are

Case underwent salpingogram for infertility with occlusion in one tubes:

A. IVF

B. ICSI

C. Clomiphene citrate✅

D. UI-H

another recall:

Case underwent salpingogram for infertility. salpingogram showed no diffusion in tubes.

A. IVF ✅

B. ICSI

C. Clomiphene citrate

D. UI-H

Here the answer is A because the tubes are bilaterally obstructed.

# This is a sum-up of female infertility due to tubes pathology:

- unilateral tube obstruction on salpingogram = ovarian hyperstimulation by Clomiphene citrate.

- bilateral tube obstruction on salpingogram = IVF

- bilateral tube hydrosalpinx on salpingogram = 1st Salpingectomy for hydrosalpinx then IVF

Paragravedous in labour, When Latent phase consedrid prolonged?

A) 2 H

B) 4 h

C) 8 h

D) 18 h✅

>20 hours for nulliparous

>14 hours for multiparous

Turbot lap Hysystrosalpipingigrogramm


Endommetriamm

-Case of female with infertility

Best way to asses tubal potency?

Answer: Hysterosalpingogram is the 1st line


x

Dysmenorrhea, how to best diagnose endometriosis:

A. laparoscopically✅

B. US

C. MRI

Pregnant lady in labor cervical dilation 4 every thing was reassuring

except there was sinusoidal in CTG for 40 minutes what to do?

A-Augmentation of labor

B-Cs ✅

C-Reassess after 2 hours

Patient P4 came for contraceptive method. Patient had regular


cycle but increase in bleeding recently. Hx of CS, and endometritis after the second CS which has
been treated. Which of the following is contraindication for IUD?

A. Hx of endometritis

B. Abnormal bleeding

C. Other things not relevant I couldn't remember

Better recall: This patient was using IUD before, but she got pregnant nonetheless. Delivered with C-
Section. Then she developed endometritis. Which of the following represent contraindication:

A- previous CS

B- history of Pelvic infection

C- history of contraceptive failure

D- genital bleeding

Abdulrahman: The answer is abnormal bleeding / Genital bleeding. UTD: Here are the absolute,
evidence-based contraindications to IUDs:

- Severe distortion of the uterine cavity

- Active pelvic infection (the patient is currently not active)

- Known or suspected pregnancy

- Wilson's disease or copper allergy

- Unexplained abnormal uterine bleeding

= Breast cancer

Female patient with ovarian tumor and during remove it the nurse notice thick secretion coming out
from the ovary, what’s the type of this tumor ?

A- Thecoma ✅

B- Teratoma

C. Fibroma

D. Granulos

Pregnant in 34 week of GA came with labor pain admitted, she had PROM 4 weeks ago , O/E cervix
is 3 cm, there is clear liquor, Ask about next step?

A- Emergency CS

B- call Anastasia and deliver pt✅

C- Wait until 37 weeks

D- give her corticosteroids

Pregnant female on vaginal examination you see abnormal cervical changes :

A- pap smear

B- colposcopy

C- biopsy

Couple going to marriage, he have siblings with SCD , he is SC trait, she is normal and have no family
history , what is the risk if they got married having children with SCD

A- very high

B- low or nill✅

C- they can't get married something like that

What tocolytic increase risk of PDA ?

A- Nifedipine

B- indomethacin✅

A 28 year old primigravidae patient delivered her baby and developed produced bleeding requ iring
1L of crystelloid, 4 units of blood transfusion and despite all medical measures patient remained
hypotensive and Hb 7.5g/dL and PLT 60. What will you administer in this case?

A-Platelets

B- Cryoprecipitate✅

Old age female [ 55 i think ] has lower abd. Mass, US showed large fibroid mass, which tumor marker
is correct ?

Ca 125 ✅

CEA

Bhcg

Ca 19-9

pregnant woman comes to the clinic concerning for lumps confined to her aureola, that is not tender,
what

is the dx?

A. Lactiferous duct

B. Mondor's disease

C. Montgomery follicles✅

married compline of blue discoloration of the cervix (Chadwick's

sign). what is the next step to do?

A. BHCG✅

Chadwick's sign is one of several physical changes that occur during pregnancy. It is an early sign
that a person is probably pregnant.

45yo Women , has abdominal distention . Imaging shows moderate ascites , pleural effusion &
bilateral ovarian masses . Ca 125 is elevated 90something . What is the dx?

A- Epithelial ovarian cancer

B- sex cord tumor✅

C- germ cell tumor

D- Forgot 4th option (something blastoma)

Meigs syndrome : ascites , pleural effusion and ovarian mass

https://t.me/obgynq/58 OB-gyn, [Oct 12, 2022 at 3:09 PM]

Bilateral ovarian mass with ascites, pleural effusion, CA 125 was 100, no description of the masses
themselves w mdri shu, then asks what is most important for diagnosing?

• Tumor marker

• Tumor histopathology ✅

• MRI of soft tissues

• Something else but it was wrong for sure

What of the Antihypertensiveis drugs are  contraindicated  in pregnancy:

A)ACEI✅

 B)methyldopa

C)hydralazine

 D)nefidinpine

Pregnant scheduled for labor induction, what to use for cervical ripening?

A. Vaginal prostaglandin✅

B. Vaginal progesterone

C. Methyldopa

D. methergine

How do you confirm the diagnosis of psoriasis in the vulva?

A-biopsy ✅

B- Clinically

8 weeks with vaginal spotting and everything else is normal and her cervix is closed. Normal
gestational sac, what are you going to do ?

A- reassurance ✅

8 weeks with vaginal spotting and everything else is normal and her cervix is closed. Normal
gestational sac, what is the type of abortion

A- threatened✅

30 years old female, pregnant, GA 33, presented with fever, rigor, and loin pain, she has

hx of UTI twice in this pregnancy, what is the most appropriate management?

A. MRI pelvis

B. Ureteroscopy

C. Septic screening ✅

D. Xray of kidney, ureter, and bladde

708- A young female on labor GA 38/40. Her pregnancy was normal, uneventful, with normal fetal
development. The labor was normal but the was difficulty delivering the placenta which complicated
with uterine inversion accompanied with vaginal bleeding 1200cc. Which of the following is the
source of her bleeding?

A- Trauma blood vessels

B- Muscular injury and laceration

C- Uterine muscles fail to contract✅

857- case of preterm labor How to confirm?

A) regular contraction on CTG

B ) digital pelvic exam dialated cervix >3cm ✅

C) hx

D) leaking fluid

Female patient with dysmenorrhea and u suspect endometriosis, what is the (most) appropriate step?

A)US if ask about intial

B)diagnostic laparoscope if ask diagnostic

C)CT abdomen

Preterm patient, necrotizing enterocolitis ,what is the strongest risk factor?

1- Low birth weight✅

2- Male gender

722- Female pregnant first trimester, which one of


the following blood chemistry will mostly appear?

A- Decrease in plasma Na

B- Decrease in uric acid

C- Decrease BUN ✅

D- Unchanged BUN

Patient presented with painless vaginal bleeding and placenta covering the internal os , Which of the
following is considerd a risk factor for her presentation ?

A- HTN

b- multigestation✅

C- smoking

D- Dm

Answer: B multigestation or multiparty

- missed abortion what to do ?

Misopristol ✅

Hypertonic saline

Myotomy

Lesion in labia majora in post menopausal female shows dysplasia (carc in situ i think) what to do

A steroid

B local excision✅

C vulvectomy

D repeat 6 months

decreased fetal kick

whats the first thing to do:

A. US

B. CTG✅

pregnant in first trimester vaginal prolapse grade2

CTG normal

vital normal what to do:

A. CS

B. Reassurance ✅

C. induction of labor

Female P7 G7 in her 30's has PPH after labour for 7 hours whats the

reason?

A- prolonged labour

B- grand parity ✅

C- multiple gestation

post menopausal scanty bleeding what’s the source :

A.uterine cervix

B.uterine endometrium ✅

C.valva

D.vagina

20 years old females, come with sever RLF pain, tenderness in ex, without fever

vital unstable :

A. Ovarian torsion

B. Appendicitis

C. Rupture ectopic pregnancy✅

218.A 23 year old lady was prescribed with azithromycin 1 gm for her

chlamydial pelvic infection. She has got a new boyfriend for the last 2

months. She has recently started contraception to avoid conception. Which

of the following contraceptive method will be affected by azithromycin ?

A) barrier

B) IUCD

C) POP

D) COCP✅

Woman her husband has gonorrhea what’s the most diagnostic for her?

A. Anogenital swap

B. High vaginal swap

C. Endocervical swap ✅

D. Urine culture

Gonorrhea: Endocervical swab

Candida: High vaginal swab

female has submucosal fibroid. She didn't complete her family yet. How manage?

A. laparoscopic myomectomy

B. hysteroscopic resection

subserosal > laparoscopic myomectomy

submucosal > hystroscopic resection

$ 60y old lady present with lower genital bleeding, she described it as Scanty and barely stain the
pad , what is the source of bleeding?

— a.Fallopian tube

— b.Ovary

— c.Uterus

— d.Genital tract

primi came for antenatal visitis she twice had small for gestational age on abdomianl examination
which indicated IUGR. Which has the most highest diagnostic valvue?

Non stress test

Fetal kick chart

Serial ultrasounds✅

CTG

Sickle cell trait, what’s the most common complication during pregnancy?

A- IUGR

B- Chest infection

C- Preeclampsia

D- UTI ✅

Sickle cell disease , what’s the most common complication during pregnancy?

A- IUGR✅

B- Chest infection

C- Preeclampsia

D- UTI

What is the hormone responsible for pregnancy?

A- Progesterone✅

B- Estrogen

C- Prolactin

Pregnant lady with Leg swelling and sign of DVT , SOB , what best next ?

A. CT PA

B. Chest x-ray

C. Doppler US of LL✅

D. V/Q scan

39 weeks of gestation, gush of fluid how Confirm


amniotic leak:

A. Methylene blue stain

B. Another stain

C. Ultrasound assessment✅

D. Microscopy of ferning cells

38 Y/o pregnant lady present with dysuria and ..lower uti symptoms. Which of the following treatment
is right ?

A-Ciprofloaxine

B-Gentamicine

ties
C-Amoxicillin- Calvunate ✅

43 yr old with inter-menstrual bleed, US showed 13 mm long mass from the endometrium, most
appropriate management?

-endometrial sample

-hystroscopy with removal of mass ✅

-US after 6 mont

-serial progestin in 3 months

woman presents with postcoital bleeding her cycles are becoming more heavy and irrigulae than
usual, she has type 2 diabetes and BMI 38 us showed endometrial thickness 18mm What is the most
appropriate next step in her management?

A) pelvic ct

B) hystroscopy

C) clinical endometrial sample ✅

60 year old with a current 5 cm fibroid, you checked her previous

ultrasonographic results before 5 years and found that it has the same size of 5 cm. What to do in
regards to management?

A. myomectomy

B. hysterectomy

C. endometrial sampling ✅

Postmenopausal —- sample

Menopause women 64 years with vaginal spotting, most management:

1_endometrial sample✅

2_follow up

3_hysterctomy

4_ unrelated

Postmenopausal —- sample

Most common complication of uncontrolled DM in pregnancy ?

Congenital anomalies✅

pregnant 31 weeks came with uterine contraction and cervix closed what to give

1. Nifedipine

2. Terbutaline

3. Indomethacin

4. Corticoseroid✅

Abruption placenta 32wks and bleeding stopped your action??

A-admission and follow up in hospital ✅

B-discharge and normal follow-up

C-high risk and follow-up with serial u/

Pregnant in 7 weeks came to ER complaining of passing tissue at home and o/e she has MODERATE
BLEEDING, what is the appropriate management?

A- Expectant management ✅

B- D&C

Other choices unreliable

I’m sur the Q like this no vitals no additional information about stability

Most common uterine fibroid location?

A- Submuocus

B- Intramural ✅

C- Sub serosal

Most common fibroid: Intramural

Most common type of fibroid that cause BLEEDING: Submucosal

Female with endometriosis not responding to OCP , what is the next step in management ?

A. Hysteroctomy

B. laparoscopic fulguration✅

1st line is NSAIDs

Not respond —> OCP

Not respond —> Laparoscopic fulgration.

Ref: Amboss.

Preeclampsia case, which one will be decrease [ no platlet ] ?

Plasma uric acid

Plasma creatinine

Plasma volume ✅

Serum urea

32 post CS 16 days , presenting with fever not responding to antipyretic and Abx, previous multiple
Cs with significant adhesion. Ct shows small bowel fistula l, which is appropriate next step ?

A.NPO and start pancreatic and gastric secretion inhibitors ✅

B. resect and anastomose

C.restrict to soft diet

D.colostomy

750- Pregnant 37 wks with PROM cervical dilated 4cm 80% effacement 0 station

High Bp and HR temperature 39.4 Ctg reassuring 170bpm

A- Dexam

B- Cs

C- Antepartum IV Abx✅

D- Spinal anesthesia

Known case of Bicornute uterus

in leopoid manouver you found globally soft structure in 1&2 grib

Fetal heart in the level of the umbilicus of the mother

Fetal kicking felt in lower abdomen

How you will deliver this baby?

✅ A) CS

Normal vaginal

C) Ventose

D) Forceps

Younge female presented with suprapubic pain and heavy vaginal

discharge, O/E tender fornices what diagnosis:

A. cervicitis

B. vaginitis

C. endometritis

D. Salpingitis✅

37 year old female pt g8 p7 at 36 weeks of gestation had previous recurrent postpartum that required
blood transfusion.

She is medically free

What is the most appropriate preventable measure in delivery?

A- deliver the patient in Operation Room

B- elective cs in 38 week of gestation

C- start active management in 3rd stage of labor ✅

D- give preload crystalloid fluid in labor to over come postpartum hypovolemia

An ultrasound picture of twins, what is the most likely cause of increased mortality or morbidity?

A-Vaginal delivery

B-C section ✅

C-Ventouse

D-Forceps?

1103- Pregnant with SCA only 2 attacks per year, normal previous pregnancy, this is her second
pregnancy came for antenatal follow up, hgb 9 next? Haemoglobin was low

A.Simple blood transfusion

B.Exchange transfusion

C. Close observation throughout pregnancy✅

70 years old female with prosthetic heart valve on warfarin, has


grade 3 pelvic organ prolapse. How to manage?

A- Pessary ✅

B- Sacrocolpopexy

C- Le Fort Technique

D- Hystrectomyg

Q8 Pregnant came with rupture of membranes and history shows herpes lesions previously what will
u do?

A speculum exam✅

B give acyclovir

C cesarean section

First exclude active lesion

If active CS

890- A 45 year old woman with 6 months amenorrhea wants to get pregnant what do order :

A- FSH and LH ✅

B- Endometrial biopsy

50 year old female, presented complaining of a sensation of pelvic pressure but no stress
incontinence. Her work involves lifting heavy objects. On examination you find mucosa bulging
through the vagina. What’s the most appropriate management:

A. Kegel exercises

B. Pessary✅

C. Bladder neck fixation

D. Colporrhaphy

Old lady 70 years old had a hesterectomty 15 years ago now she has a very large vaginal vault
prolapse which covers something I don’t remember and reach the premium what is the appropriate
treatment?

A- vaginal pessary ✅

B- colpoclesis

C- anterior posterior repair

D- vaginal surgery something

Para 4 urinary frequency with no leak ,work as loading heavy boxes to her shop , on exam there's a
large bulge on the anterior vaginal wall.

1) Kegel exercise

2) Vaginal pessaries✅

3) anterior colporrhaphy

Initial pessaries

Definative will be anterior colporrhaphy

what is the management of women who came with moderate cystorectocele?

A. Ant and post Colporrhaphy ✅

B. post Colporrhaphy

C. Ant Colporrhaphy

D. Manchester procedure

70 years old woman, sexually active, hysterectomy done before, and she complains of

prolapse and enlarged vaginal opening, what is the appropriate procedure?

A. sacrospinous fixation ✅

B. Ant and post colpoperineorrhaphy

C. Manchester repair (Fothergill operation)

30's years old women P6 (not mention complete her family) come with second degree pelvic organ
prolapse, also have cyctocele and rectocele , what is most appropriate management ?

- vaginal hystrectomy

- anterior colporraphy

- posterior colporraphy

- Manchester repair✅

-Cystocele only = anterior colpoperineorrhaphy ✅

- Rectocele only= posterior colpoperineorrhaphy✅

-cystorectocele= Ant&post colpoperineorrhaphy✅

- both of them + uterine prolapse=fothergill's operation ( manchester)✅

Long case mentioned Pseudohyphae and ask about management?

Oral fluconazole✅

make sure she is not pregnant in the q

If the recall change and she was pregnant the answer will be Topical azoles or nystatin

Treatment for Bacterial vaginosis?

Metronidazole ✅

Pregnant women came with abdominal pain and vaginally bleeding denied pass tissue,

cervical os closed, US show intrauterine sac and anembryonic most likely Dx?

A. Inevitable abortion

B. Threatened abortion

C. Pregnancy unknown location

D. Pregnancy unknown viability✅

Female in labor (+34 weeks for sure, can’t remember exactly), in prenatal Hx she had UTI (Klebsilla)
and asthma for which she takes ventolin. what to give during labor and delivery?

A- Oxytocin

B- AB’s ✅

C- Steroid.

D- Mg sulphate

Which of the following is true regarding single umbilical artery?

A) Associated with maternal diabetes ✅

B) Major malformation in 80% of cases

C) Low fetal mortality

D) Associated with african race

Pregnant come with vaginal bleeding medical student want to do physical examination what you well
do ?

A) Let him do examination

B) refuse because there is no need

C) tell him we should do US first to roll out placenta previa ✅

43 years old multiparous female with intermenstrual bleeding. On ultralsound there was long
echogenic structure in the endometrium. What is the most appropriate management?

A) Hysteroscopy and removal of that structure. ✅

B) Endometrial sampling

Case of pregnant 16 weeks developed right side abdominal pain (did not specify upper or lower).
Temp 73.8

Most common cause

A) gastritis

B) pancreatitis

C) cholecystitis

D) appendicitis✅

Doctor while taking a history, He found history of severe dysmenorrhea

What would be likely the cause?

A-Ovarian thecoma

B-Ovarian fibroma

C-Endometriosis✅

D-Leiomyoma

Pregnant 38wk at hospital planned for elective CS , US showed “

Placenta ATTACHED DEEPLY “ into uterine wall , what’s Dx:

A- P. Increta

B- P.Accreta✅

C- P. Penceta

D- Placenta previa

Delivered 4 months ago and now she came with positive pregnancy test at home and forgot her
LMP, accurate way to estimate her EDD?

A- ultrasound in 18 weeks

B- vaginal ultrasound ASAP in first trimester✅

C- when the mother feel fetal movement

Pregnant lady and US showed polyhydramnios. Which of the following is associated with this
condition?

A) Bilateral renal agenesis

B) Trisomy 21 ✅

C) IUGR

Young female presented to the ER complaining of general fatigue symptoms and dizziness and
mention that she is bleeding for 15 days since her period started.

No mention of Examination or current bleeding status.

Lab: Hb 7

What is the most appropriate next step?

- US

- Start blood transfusion ✅

- Stop the bleeding and send her home

Monochorionic and Diamniotic twin what is the timing of cleavage ?

A- <72 hour

B- 4-8 day ✅

C- 8-12 day

D- >13 day

$ What is the time interval between ovulation and cleavage in dichorionic diamniotic twins?

— A. 0-3 days
(0-72 h)
— B. 4-8 days

— C. 9-12 days

— D. >12 days

A 28 year old primigravidae patient delivered her baby and developed produced bleeding requiring
1L of crystelloid, 4 units of blood transfusion and despite all medical measures patient remained
hypotensive and Hb 7.5g/dL and PLT 60. What will you administer in this case?

A. Platelets

B. Cryoprecipitate✅

A 20 years old pregnant known case of Idiopathic Thrombocytopenic purpura. Her deliver was
complicated by post partum hemorrhage. What will you give her next? A. Packed RBCs B. Fresh
frozen plasma C. Cryoprecipitate D. Plateletstransfusion✅

ITP = platlet

Patient with PPH and she is known case of ITP , treatment?

A. Cryoprecipitates

B. FFP✅

ITP = platlet (if not coose FFP)

Pregnant developed abruptio placenta and 4 prbc given and still she is hypotensive with low platelet
and D dimer 800 , which of the following is the appropriate managment?

A- cryopreciptate ✅

B- platelets

Hormone that confirm ovulation

Estogen

Progesterone✅

Fsh

Lh

pregnant in 31 weeks presented to ER with labor pain, 4 weeks ago she was diagnosed with
PPROM, now the the amniotic fluid is exposed, and she has 3 contractions in 10 min. What is the
next step in management?

A- High vaginal swap for culture and sensitivity ✅

B- IOL

C- Augmentation of labor

D- Fetal blood sampling for ph and liquor

Patient just gave birth, started to bleed heavily, what to do Next ?

A) call for help and ensure cross matching ✅

B) start blood transfusion w/ O- blood

Patient had ruptured a membrane 1 weeks ago, now she is 31 week pregnant with labor pain.
Examination showed liquid liquor. What to do?

A- swab for GbS culture ✅

B- induction of labor

C- augmentation of labor

D- C section

$ 21- Female primigravida with irregular cycle and she is infertility for 3 years and know with
vaginal spotting and tender abdomen and tender in cervix motion what to do?

— A- Confirm pregnant status

— B- US

— C- Ask about cause of Infertility

$ 19 -20 yrs female c/o abdominal pain, vomiting, constipation for 2 days... what do for her?

— A-Inr

— B-Cbc

— C-Bhcg

— D-Urine analysis

$ 20- weeks since lmp, no symptoms, with fundal height slightly above symphysis pubis, highly
valuable investigation:

— a. US

— b. Chest abdomen ct

— c. abd x-ray

$ 22---years old patient, primigravida with spontaneous abortion. Asking you to address her
concern about her next pregnancy what to do?

— A-tell her spontaneous abortions can occur 10-15% of the time

— B-Tell her never to get pregnant 1

— C-Do a full workup for reccurent pregnancy loss

$ 24-. risk for miscarriage increase with ??.

— A) Maternal age

— B) Gestational age

— C) Pre existing conduction HTN and diabetes

$ 26-. two questions about mutiple miscarriages asking about the probable diagnosis?

— A) antiphospholipid syndrome

— B) scleroderma

— C) arthritis

female prenatal visit history of hypothyroidism on Levothyroxine , it's controlled and T4 tsh level are
normal , no mention in Q if she is pregnant or not , just prenatal visit

-increase dose✅

- continue same dose

- decrease dose

women with hypothyroidism. she is in levothyroxin

she get pregnant with control of her hypothyroidism ( they put a no. of TSH and T4 within normal
both of them ) next step

A- increase the dose ✅

B- decrease the dose

C- don't change the dose

women presented with labor and was complaining of continous abdominal pain that was worseing.
There was no bleeding. Pelvic examination showed 6 cm cervical dilatation. what is the next step:

A-US.

B. CTG.

C. Repeat pelvic exam

I think CTG I am not sure

‫نشوف وضع البالسنتا اول‬

A woman with submucosal fibroid that is causing her infertility what is the most appropriate
treatment?

A- Hysteroscopic myomectomy✅

B- laparoscopy myomectomy

C- laparotomy myomectomy

D- laparoscopic hysterectomy

A lady with history of infertility normal labs normal semen analysis of the husband she did
laparoscopy and found to have bilateral tube obstruction, management?

IVF

pregnant at 30 week with history of clear fluid discharge, CG shows regular continues contraction,
what confirm that the patient is having preterm delivery?

A-regular contraction of CTG(not sure)

B- history alone is enough to confirm

C-If there is no passage of clear fluid then it is not preterm

D-DIGITAL exam to see cervix length✅

32 y/o Obese woman complaining of irregular mensuration and chronic anovulation, endometrial
sample showed: atypical complex hyperplasia .. what’s the most definitive treatment(complete
scenario) ?

- tamoxifen

-Letrozole

-progesterone ✅

-spironolactone

Pregnant with large genital wart how to manage:

A- Cryotherapy ✅

B- Electrocautery

C- Substance to put on it forgot the name

Case about women have (vesicles painful) around genitalia, What is the cause?

A. Varicella

B. Hpv

C. Herpes ✅

high Bmi, hirsutism, cycle abnormalities“ dx written as pcos“ she doesn’t want to be pregnant tx?

A- Metformin

B- OCP✅

C- spironolactone

Fetus with breech presentation flexing hip and extending knees, what's the presentation?

A- Frank breech✅ .

B- Complete breech

C- Incomplete breech

D- longitudinal

$ A pregnant woman not attending to visits came at 38 weeks in labor , fetal heart beat is felt at
umbilical level with flexed hips and extended knees ?

Anonymous Poll

— A- Frank breech

— B- Incomplete breech

— C- Complete breech

24-fetal spine parallel to mother and head at the fundus, the knee are extended and the hip is
Flexed , what is the presentation?

A-Complete breech

B-incomplete breech

C-frank breech✅

Female in her twenties presented with a small mass in the vulva, it’s

smooth, shiny and round. Upon local incision it revealed jelly-like

secretions. Dx?

A- Vulvar SCC

B- Vulvar mucinous cyst✅

C- Hidradenitis suppurativa

Nurse asked you what is true about CTG:

A- fetal head compression - late deceleration

B- fetal head compression - variable deceleration

C- cord compression- late deceleration

D- cord compression - variable deceleration✅

Female patient presented with menorrhagia and a 10 cm fibroid. The doctor decided on a
hysterectomy but she declined as she want to preserve fertility. What is the most appropriate
alternative management?

A. Oral progesterone

B. Myomectomy✅

C. Uterine artery occlusion

Female regular period women want to conceive. What is the laboratory indicative of ovulation?

A- Urine FSH LH✅

B- Us

C- Progesterone 21

D- Estrogen level

-Pregnant with high random blood sugar reading and fasting sugar. She’s at risk of what vaginal
Infection?

A. Candida✅

B. Bacterial vaginosis

-Pregnant patient with polyydrominous and fetal bradycardia ?

What is tha cause of bradycardia

A.Cord prolapse

B.Placenta abruption / placental insufficiency ( cant remember which one)

C.Placenta previa

D.Congenital heart disease

Another recall :

Pregnant 38 wk with polyhydramnios, presents with rupture of membrane and severe abdominal
pain, CTG show fetal bradycardia, whats the diagnosis :

- vasa previa

- cord prolapse

- placental abruption✅

- can't remember

Answer: B by earth

Another recall

A pregnant lady in her 38/39th week comes to you complaining of abdominal pain and vaginal
bleeding, she has a tender and tense uterus, the fetal CTG demonstrated prolonged fetal brady
cardia, whats the Dx?

a. Cord prolapse

b. Placental abruption*

c. Placenta previa

d. Vasa previa

Answer : B

Earth % &

40 weeks gestation G3 p2 presented in labour ruptured membrane 5cm cervical dilatation for 3h 60
% effaced head station - 1. Baby non stress test reassuring mother good vitals What is the
appropriate plan of management?

A- urgent c/s

B- observation✅

C- augmentation of labour by oxytocin

D- other non relvant

Which pf the following signs considered high diagnostic value for RH mother in ultrasound?

A-Fetal Bradycardia

B-Fluid around fetus

C-Thin uterus

D-MCA diastolic flow "something like this not sure". ✅

Primary infertility with scanty irregular cycle and very high FSH, most important work up?

A-FSH/LH in urine I think

B-CBC

C-Pelvic US ✅

D-chromosomal analysis

25 weeks pregnant only abdominal discofmort cervix short soft cervix 1 cm and some of membrane
out from cervix , what is the dx?

A- cervical incompetence✅

B- not remembered

IUGR most common risk factor

A- age

B- smoking

C- HTN✅

febrile patient PPH 2 weeks what’s the cause :

A.Endometritis✅

B.retained product of the conception

60 years old lady presented with lower genital bleeding, she described it as

scanty and barely stains the pads, what is the source of bleeding?

A. fallopian tubes

B. ovaries

C. uterus✅

D. genital tract

39 female asymptomatic with 5 cm fibroid, most appropriate management ?

A- myomectomy

B- biopsy

C- hysterectomy

D- clinical surveillance✅

27 year old complaining of 9 months amenorrhoea  investigation

LH=32

FSH=51

What is the cause of her presntation:

Pcos

✅ Premature ovarian failue

Ovarian cyst

Pregnant diabetic at 20 week gestation the anatomical screening was normal, now at 34 week and
the fundus height is 38 cm, what explain the difference between the gestational age and the size?

A-polyhydramnios✅ (not sure)

B-undetected twin pregnancy

Asymptomatic pregnant asked for

urine culture for some job reason, what is the best time to get urine culture for asymptomatic
bactururia?

A- 12✅

B- 20

C- 26

D- 32

Pregnant preterm PROM 33 wks or less(i cant remember) ttt consrvative then after 3 days fever and
abdominal pain only what you do?

A..Delivery✅

B..Search cause of fever then treat it

Mother found to have excessive fluid. (They mean polyhydominaas ). Which of the following
considered high risk for developing her condition?

A- Uncontrolled DM ✅

B- Renal Agenesis

C- IUGR

Post partum hemorage, which of the following is used to stop bleeding or most app.

A- Misoprostol ✅

B- Propfol

C- Mg sulphate

Dm and polymenorrhagia

What the most appropriate investigation ?

A.Endometrial biopsy ✅

B.US

42 femal, last 15 months heavy manistral bleeding its progress, uterus is normal and ovary not
palpable ,us 3-2 lesion

A- adenomyosia

B- submucus liomyoma

C- DUB

41w pregnant ctg twice weakly Ctg show decceleration What is the highest value

1- delivery

2- ctg daily -

us for bpprofile -

us for amniotic index

pt with hx of ectopic pregnancy, now pregnat. asking about the risk of geting ectopic?

A.5%

B.20%

(no any choice of 10%)

A mother coming for the first visit with an Rh-positive father and a baby and a mother with an
unknown RH status, what is the next most appropriate step?

A. test the mother

B. amniocentesis

60 yo women presented with intermittent vaginal bleeding , no weight loss, unremarkable past hx,
what is the most likely diagnosis?

a- endometrial cancer

B-endometrial hyperplasia,

c-endometrial atrophy

Pregnant 8 wk, known case of cervical incompetence

A- Cerviclage now

B- Cerviclage at 14-18 wk

C- Progesterone

D- Nothing

Woman rh negative, gave birth to rh+ baby, she took anti-D in the 28th week GA. What to do now?

- 300 micro gram ✅

- 1000 micro gram

- 2000 microgram

- no need

A mother coming for the first visit with an Rh-positive father and a baby and a mother with an
unknown RH status, what is the next most appropriate step?

A. test the mother ✅

B. amniocentesis

23 wife medically free, regular cycle, c/o primary infertility for 3 months, Husband 26, medically free,
has 1 cousin autustic, 1 with trisomy 21, Most appropriate :

A- continue trying ✅

B- semen count

C- gentic testing

3 months only

35 y/o, primary infertility for 3 years, has 3 cycle of clomid as a case of polycyctic ovary, Husband
smoker.

A- semen analysis ✅

B- husband prolactin

C- 2 day of cycle hormons

D- hystosalpingo

Female recurent uti , she noticed its always after intercourse , what to do :

A ) give Abx if symptomatic

B ) give Abx course for 6 months

C) give ABx course for 2 weeks

D ) give prophylactic Abx after intercourse✅

Women whose recurrent UTIs are associated with sexual intercourse should be offered postcoital
prophylaxis. This involves taking a single dose of an effective antimicrobial (eg, nitrofurantoin 50 mg,
trimethoprim-sulfamethoxazole

[TMP-SMX] 40/200 mg, or cephalexin 500 mg) after sexual intercourse. (Medscape)

3- Female patient pregnant with history of previous 4 C-section presented with mild bleeding and
hypotension, what is the most likely cause of her presentation “This is the full scenario no further
details”

A- cord prolapse

B- Uterine rupture ✅

C- abruptio placentae

D- fetal vessel rupture

26-year-old female came to the Clinic complaining of severe lower abdominal pain tat started just
before her menses and disappeared on the 3rd day, the pain so severe that affect her daily activity
and prevent her from going to work for many days. Her boss is not happy.

Which of the following is the most appropriate management?

- hysterectomy

- Paracetamol

- OCP✅

No NSAID in the options

Pregnant with low risk pregnancy at 32 weeks came with vaginal bleeding, diagnosed as placenta
abruption and managed conservatively , most appropriate next step?

A- remain in hospital✅

B- reclasify as high risk and f/u with US for fetal growth

C-apply as low risk with f/u outpatient clinic

D-apply as low risk and f/u with US for vaginal bleeding

Twins both in vertical position with normal heart how to deliver ?

A ventose

B Forceps

C VD✅

D CS

Female patient with vulvovaginal psoriasis, how to confirm diagnosis

A-clinical exam

B-biposy✅

C-scraping skin edges with KOH

Incapacitating symptoms prior menstrual cycle ttt?

A- Ssri

B- Ocp✅

Female had infertility for 6 years, complains of severe dysmenorrhea

-lyomyoma

-endometriosis ✅

-adenomyosis

Female, p6, feel heaviness and discomfort on her pelvic region, exam showed mass coming from
introitus , dx?

A- Rectocle

B- Cystocele

C- Uterine prolapse ✅

D- Enterocoele

Typical case of preeclampsia, asking about which of the following clinical findings with this
condition?

A-elevated mother liver enzymes

B-oligohydramnios ✅

C-polyhydroamnios

Bicornuate uterus, breech presentation CTG normal

A. Svd (spontaneous vaginal delivery)

B. cs✅

large fibroid what to give to decrease size before the surgery?

A- GNRH✅

Patient (forgot her age), P4 (i m not sure), K/C/O PCO and a smoker, which of the following increases
her chance of endometrial cancer?

A-PCO ✅

B-Age

C-Smoker

D-Parity

45 years old asymptomatic patient, on pelvic ultrasound she was found to have uterine fibroid at the
fundus measuring 2*3 (not sure about size ) what is the next best appropriate plan of management

A. Urgent hysterectomy

B. Urgent myomectomy

C. Follow-up with yearly ultrasound✅

D. Follow-up with serial ultrasound and CBC every 2 months

WHAT is the best indicator of the severity of post-partum hemorrhage?

A.Hct

B.Hg

C.Amount of blood loss✅

•Pregnant at 38 weeks, found that the fetus has intrauterine growth restriction, vertex presentation,
what is best choice for delivery:

-SVD ✅

-CS

-Ventos delivery

24 y.o Patient high FSH, LH, normal GTH, Amenorrhea for 9 months. What is diagnosis?

A. Ovarian tumor

B. Ovarian insufficiency✅

C. PCOS

Women with 42 weeks gestation No contraction , cervical dilation 2, -3 station, normal CTG, what u
want to do:

A- reassurance

B- Cs

✅ C- induction of labor

D- augmentation labor

Molar pregnancy follow up of HCG:

A. Weekly till 3 reading negative then 6 months✅

B. Weekly then monthly for 6 months

C. Monthly then weekly

D. Monthly till negative

Female pregnant with recurrent late deceleration and dilated 6cm what to do ?

A. Emergency CS✅

B. Reassure

C. Induction

Pregnant in 38 week gestation presented with labor. during labor and the fetal head is ingaged at 0 or
+2 station (i think) you noticed cord with head in cervix which disturb labor.

Which of the following is the most appropriate management?

- oxytocin

- CS✅

- Ventous

- Forceps

How to differentiate between hypovolemic shock and postpartum hemorrhage ?

A. Hemoglobin

B. Hematocrit

C. Increase pulse✅

With UPT positive at home now presents with lower abdominal pain. Pain worsens on cervical
touching. What u ll do initially.

A. Transvaginal assesment✅

B. Deep palpatiom of pelvic to rule out masses

Pregnant with SCD Hg 9 and hemolysis features in labs , how will you manage?

A. Start hydroxyurea

B. RBC replacement✅

C. Plasma exchange

D. Blood exchange

Patient has history of amenorrhea for 6 weeks presented with abdominal pain on examination. Is
there fluid on Douglas pouch & clot blood?

Answer: Rupture ectopic pregnancy

A dr found a blood in pouch of Douglas?

A. Exploratory laparoscopy✅

B. Endoscopy

C. Conservative

What of the following will cause oligohydramnios?

A-Placental insufficiency✅

B- DM

C- Chriongioma

D-Dudenal atresia

After Vaginal delivery . Pt had vaginal bleeding . doctor see multiple venipanctur bleeding . Which of
following is appropriate management:

A. Coagulation correction✅

B. internal iliac ligation

C. paking the uterus

D. B lynch

A patient with postpartum hemorrhage who was resuscitated then they found that there is persistent
bleeding at several puncture sites what is the next important step?

A. Reversal of coagulopathy✅

B. Oxytocin C. Prostaglandin

Type of the estrogen in pregnancy?

A. Estriol✅

Female pregnant GA 20 something present with complain of uterine contractions CTG done showed
acceleration and 4 contractions in 10 minutes, what is the appropriate management?

A. Reassure and discharge

B. IOL

C. Check cervical dilation✅

D. Tocolytics (not sure if this was an option)

55 years old female menopause present with heavy vaginal bleeding associated with weight gain BMI
35. TSH done was normal

Endometrial biopsy showed adenomatous hyperplasia

What is the pathophysiology?

A. Adrenal hyperplasia

B. Peripheral conversion of precursors to estrogen✅

C. Genetic mutation of endometrium

$ Mother delivered twins 2 hs ago now having bleeding cause?

Anonymous Poll

— Atony

— Bleeding disorder

— Trauma

— Retained tissue

$ Young patient with infertility and irregular menstural cycles.

Labs:

FSH within normal or low slightly LH normal Prolactin very high 2500 I don’t remember if TSH
mentioned or not What is the most appropriate management?

Anonymous Poll

— Clomid

— IVF

— Bromocriptine

— Intrauterine insemination

After hysterectomy urine out of the vagina:

A. Uretrovaginal fistula

B. Urethrovaginal fistula✅

C. Vesiclovaginal fistula

q about the mechanism of teratogenecity of ACE inhibitors in pregnancy

renal agenesis✅

Gestational diabetes is ?

A. Normal pregnancy associated with diabetogenic condition due to progesterone

B. Normal pregnancy associated with diabetogenic condition due to estrogen

C. Normal pregnancy associated with diabetogenic condition due to FSH

D. Normal pregnancy associated with diabetogenic condition due to lactogen✅

$ Female with polyhdromanis you will consel her about potential associated with

Anonymous Poll

— Trisomy 21

— Renal agensis

— Uterine rupture

— Fetal restriction

A 29 years old female pregnant at 29 week GA came to the ER with lower abdominal pain, during
Ultrasound you detected 100ml of blood behind the closed cervix. What would you give:

A. Dexamethasone✅

B. Indomethacin

C. Mg sulphate

D. Nifedipin

$ Lichen’s sclerosis associated with which valvular cancer?

Anonymous Poll

— A) SCC

— B) BCC

— C) Adenocarcinoma

— D) Malenoma

8- PPH post SVD, oxytocin & massage was done, what NEXT?

A/ Hysterectomy

B/ B lynch suture

C/ Embolisation

D/ Barik Balloon✅

Healthy lady with Preterm labor what to give her?

A. Indomethacin ✅

B. magnesium sulfate

C. Nifedipine

D. Terbutaune or nitroglycerin

pregnant in labour on at term. OE (describe breech . As head in funds and filix both hip and knee )
spine pf baby parrele to spine of mother. What is the lie.( Atention ask for lie not presentaion)???

A.longitudinall✅

B.transverse.

C.breach

D.cephalic

38 weeks pregnant presented in labor pain with progressive cervical changes. In the labor room, she
had vaginal gush of fluid:

A. Spontaneous rupture of membrane✅

B. PROM

C. PPROM

Pregnant with colonization of GBS and admit for labour , which of the following increase risk of
neonatal infection ?

DM mother

Twin

Preterm ✅

Rupture of membrane > 6hrs

Patient with PCOS wants to get pregnant. How to manage?

A. OCPs

B. Metformin, weight loss, and Danazol Case ploy cystic ovary doesn’t want to get pregnant
treatment?

C. Metformin, weight loss, and Clomiphene

Cm

Time of ovulation:

A. 36 hours after LH surge✅

B. 36 hours before LH surge

C. At time of LH surge

D. Something related to estradiol

45 yrs old female with hx of secondary infertility “already has children”, her menses associated with
menorrhiga and pain, your diagnosis
Pt had hx of D/Cs
A. Adenomyosis✅

B. Endometriosis

C. Pcos

38 y/o female at her 20th week of gestation presenting with vaginal spotting with no hx of trauma her
blood group is O-, she is vitally stable. US showed normal size fetus with long cervix. What will you
do for her ?

A. Give anti-D✅

B. Discharge home

C. observation

D. continue on her multivitamins

32 GA came with preterm labor then had preterm rupture of membrane and was given Abx and
steroids. What to give

Nifedipine✅

Mg sulfate

Pt with dysmenorrhea and multiple pregnancy. One of her pregnancy was delivered by CS and all of
them was induced. Examination normal uterus not enlarged not tender and no adenexial mass

Endometriosis

Adenomyosis

Fibroid

pelvic congestion syndrome✅

45 year old female 6m misssed period She is not pregnant she want to know what is the most
valuable and significant lab for postpartum :

A. LH✅

B. Cortisol

C. Calsitrol

D. Testosterone

Case of multi para with prolonged delivery for 5 hours and they use a forcebs to deliver a big baby
4.5 kg

She developed postpartum hemorrhage:

What is the cause of this bleeding

A-prolonged lapor

B-use of forceps

C-big baby ✅

30 years old female 30 weeks GA 2cm cervical dilatation 50% effacement with PPROM the doctor
give her steroids & tocolytic drugs, Tocolytic purpose ?

A. Delay labor until steroids work

B. Prolong pregnancy until 37

The lady was G2 P1, her labor was 2 hours long (baby came out before she barely made it to the
hospital). Baby’s weight was 3000. She started bleeding after delivery of the placenta. Question
asking about the cause of PPH.

A. Baby’s weight

B. Prolonged labor

C. Precipitous labor✅

D. Large multigravida

Pregnant has RUQ, US was was done and was deemed to be CHOLELITHIASIS, Next step ?

A. Lap chole

B. Open chole

C. Conservative management and reassess after pregnancy✅

weeks in labor with Intrauterine fetal death with regular contraction and cervix 6cm, mgt ?

A. CS

B. Augmented labour✅

42 weeks in labor 7 cm dilated, meconium staining, regular and strong contractions CTG 100 fetal
hearts?

A. C/S✅

B. Augmented labor

42w IOL w prostaglandins contractions started after an hour fully dilated +2 but sudden fetal
bradycardia 70 and keep getting worse mx?

A. C/s

B. Start pushing and consider ventose✅

C. Give oxytocin

Female has asymptomatic fibriod i thing and the laps shows > Hg 60, normal [ 120-140] Aske what is
your next step in management?

A- Treat the anemia [ written like this ] ✅

Female Asymptomatic fibriod best managment?

A- Follow up with pelvic US yearly ✅

B- Follow up with pelvic US and CBS

[ or another test not sure ] every 3 months!!

Molar pregnancy case treated by dilatation and suction. What is the MOST COMMON early
complication?

A. Perforation ✅

B. Pneumonia

C. Amniotic fluid embolism…

According to UTD Uterine perforation — is the most common immediate complication of D&C.

Common complication AFTER uterine Evacuation?

A. Perforation

B. Infection✅

C. Bleeding

D. Abscess

Uterine perforation is the most common immediate complication of D&C.

After the procedure -> Infection (rare), intrauterine adhesions

Most common complication of hysteroscopy?

Bleeding

Perforation ✅

Infection

Most common complication of hystroscopy?

A-infection

B-hemorrhage✅

C-adhesion

Explanation: uterine perforation is the most common complication, followed by fluidoverload then
hemorrhage.

case of young women who came complain of sudden sever abdominal pain for 12 hours, she had
history of embryos transfer 4 weeks ago. was hypotensiove

A - ovarian cyst

B - ectopic ✅

pregnant with multiple congenital anomalies abortions. When can you do detailed investigations?

A- 442288 (literally it was typo in exam and I freaked out. i was looking for 16-18 but nothing to be
found)

B- 14-16

C- 18-20 ✅

D- 22-24

Asymptomatic newly married female pt came for the clinic for general gyn evaluation

What is the highest diagnostic test?

A)General appearance

Histortation
B)Digital pelvic exam

C)Abdominal examination

D) history ✅

Exam

pregnant present as placenta abruptio + DIC + IUFD

Her cervix is 7cm

What is your management

1)expectant

2) augmentation

3)CS ✅

4)D&C

Pregnant lady 35 weeks gestation Known case of Bicornute uterus in leopoid manouver you found
globally soft structure in 1&2 grib Fetal heart in the level of the umbilicus of the mother Fetal kicking
felt in lower abdomen How you will deliver this baby?

A) CS ✅

B) Normal vaginal

C) Ventose

D) Forceps

93 most predictable complications of pregnancy:

A- Abdominal pain

B- Ankle edema

C- Vginal bleeding

D- Heartburn ✅

Another recall:

case about recurrent pregnancy loss then ask “ most preidictable complications of pregnancy:

A- Abdominal pain

B-Ankle edema

C- Vginal bleeding ✅

D- Heartburn

If asking generally D

If mentioned Hx of miscarriage C

Female present with vomiting, she was used female female condom , I think upon Speculum vaginal
examination there is dusky discoloration on the cervix .. which of the following is the next step :

- pelvic US

- B-hcg ✅

-huhner test

Pregnant with history of uterine fibroid, 34 gestational age (not sure) presented with abdominal pain
and fever. Examination unremarkable with closed cervix no passage of tissue. Which of the following
is the most appropriate management??

A. Observation ✅

B. Cesarean section

C. Induction of labor

D. myomectomy

37 weeks pregnant pt with placenta totalis present mild vaginal bleeding, when to admit her for
labor?

A- immediately ✅

B- after planning CS

B- after planning CS

C- No need to admit

D- after another episode of vaginal bleeding

$ Hx of asthma in labor with pph doctor did massage but didnt help she stil in bleeding Which of
the following contraindications is the most appropite managment to her at this stage ?

— Oxytocin

— Misoprostol

— Methylergonovine

— Carboprost

Female take rubella vaccination and get pregnant (less than 1 month).

What is prognosis

A- Favorable pregnancy outcome ✅

B- fourfold malformation C- increase risk of pregnancy

Female post term 42 weeks indication of labor had been started what will you give her?

A-steroid

B- prostaglandin✅

Pregnant lady in labor cervical dilation 4 everything was reassuring except there was sinusoidal in
CTG for 40 minutes what to do?

A- Augmentation of labor

B- Cs ✅

C- Reassess after 2 hours

Pregnancy typical dvt symptoms then developed PE sx, how to dx?

A- Ct

B- D dimer

C- V/Q

D- Us for lower limbs ✅

38 weeks stable head +3 ctg showing deceleration ?

A-c.s

B- mechanical delivery ✅

US detailed anatomy scan :

A-18-20 wk✅

B-10-12 wk

C-30

Best indicator of chromosomal abnormalities in which week?

A-16 to 18wk ✅

B-18 to 22wk

C-13 to 16wk

D-24 to 28wk

lady mised period which most appropriate time of us

A. 11-13 weeks ✅

B. 16 - 18 weeks

C. 18 - 22 weeks

Female pregnant first trimester, which one of the following blood chemistry will mostly appear ?

A. Increase in plasma Na

B. Decrease in creatinine ✅

C. Increase BUN

D. Unchanged BUN

1- Case 34 years smoking and HTN risk factor of vaginal bleeding with pregnancy

A- age

B- HTN

C- smoking✅

$ RLQ pain ..

No fever , normotensive , pregnancy test is -ve On imaging There is mass 7 cm size

— Appendicitis

— Ectopic pregnancy

— Rupture cyst

— Ovarian torsion

A 25 year old primigravida at 36 weeks gestation presented with

feeling fatigue and unwell. She shortly developed shortness of breath and

died. XRAY showed Enlargement of the cardiac silhouette What is the

likely cause of her presentation?

A) Amniotic Fluid Embolism

B) Peripartum Cardiomyopathy ✅

38 GA presented with headache and high BP. Labs show Hb 120 (within

normal range), low platelets, high liver enzymes. How to manage?

A- immediately delivery ✅

B- MgSo4

Best test to see the glycemic control during pregnancy in the routine visit in pregnant with DM?

- random glucose

- fasting

- hba1c ✅

58 yo female presented with pelvic organ prolapse and Her BMI is 38. She has a family history of
surgical repair of POPs. What will you most commonly find ?

A- Cystocele ✅

B- Rectocele

C- Enterocoele

D- Uterine prolapse

Another recall: 48 y.o very obese lady, BMI 41 had hx of pelvic organ prolapse had no surgery, but
she had + family history of prolapse repair what do you except she’s having now:

A- Enterocele

B- Rectocele

C- Uterine prolapse

D- Cystocele ✅

48 Y/O very obese lady,BMI 41 had plevic organ prolapse many time and then had surgery, what do
you expect she’s having now :

A- Entrocele ✅

B- Rectcele

C- Uterine prolapse

D-Cytocele

E-Vesicocele

48 Y/O very obese lady,BMI 41 had plevic organ prolapse many time and then had surgery, what do
you expect she’s having now :

A- Entrocele ✅

B- Rectcele

C- Uterine prolapse

D-Cytocele

E-Vesicocele

After bariatric surgery , what expect

Cystocele✅

Rectocele

Enterocele

Pelvic wall surgery —- enterocele

Any other surgery choose the most common (cystocele)

If the patient is complaining of constipation, think of rectocele.

30 y/o Pregnant 30 gestation came with severe antepartum hemorrhage, CTG with deceleration, the
mother BP 90/60 what is the appropriate next step:

Steroids

Observe

CS ✅

pregnant woman came with typical symptoms of DVT then she developed SOB in duppler US you
found nothing what is your next step?

A- CXR ✅

B- CT

C- v/q not sure

Pregnant came to antenatal clinic for folow up, her abdominal is not consistent with fetal gestation
fetal growth restriction was detected. How to follow up with patient?

A- Serial ultrasound ✅

Old lady medically free with difficulty defecation and constipation, during defecation she introduce
her finger in the vagina to intiate movement, management?

posterior colporrhaphy ✅

Anterior colporrhaphy

Enterocele resection

Hystrectomy

Pregnant in 30g with recurrent UTI :

A- Ct

B- Ultrasound ✅

C- Culture

Another recall: Pregnant with recurrent UTI what to do?

A- X ray

B- US ✅

C- ureteroscopy

D- cystoscopy and RET's

recurrent pregnancy loss in 1st. Trimester with Hx of theomebolism

A- warfarin life long ✅

B- enoxaparin life long

C- aspirin life long

D- fandiprix life long

-female had GDM with her second pregnancy which was 6 months ago with significant family history
of DM2, comes today for screening

Fasting = 7.1

HbA1c = 5. 2

-all in normal-

What you will give her:


OGTT?
-insulin

-begunide ✅

-other drugs

Patient 45 presents with menorrhagia. US reveals huge fibroid

distorting endometrial lining. What's the most appropriate step to do?

A. Hysterectomy ✅

B. Artery ligation
hysterscopic myomectomy?
C. Endometrial ablation

Pregnant I forgot which week but the was <34 week with abruptio

placenta with minimal bleeding that was control and she is vitally stable ?

What is next

A ⁃ Stay in the hospital ✅

B ⁃ Discharge and f/up closeley

C⁃ Immediate delivery

$ 37 years old woman asymptomatic

found to have 5x7 subserousal fibroids

Most appropriate step in management?

— Myomectomy

— Hysterectomy

— follow up with US every 12 months

— follow up with US and Complete blood count every 2 months

$ 47 year women came to the clinic complaining of

abdominal distention and heaviness

US and CT : Ovarian mass 10*12cm , ascites and thicking of peritoneum

— Surgery only

— Surgery with chemo

— Surgery with radiation

— Radiation therapy

What of following decrease the risk of preeclampsia ?

A- ASA ✅

“Aspirin- acetylsalicylic acid “ASA”

$ Pregnant at 24 week all is good she’s just complaint of protruding mass

from the vagina, diagnosed as posterior vaginal wall Prolapse, what to do ?

— reassure

— Emergency delivery

$ 30 year old female pregnant with new symptoms of dyspnea,

productive cough and fatigue. Doppler US was negative what is the next

test to do:

— CTA

— D dimer

— spirometery

— ventilation and perfusion scan

$ Which of the following associated with increase risk of endometrial cancer?

— nullparty

— underweight

— oral progesterone

— family hx of thyroid cancer

$ which valvular heart disease has the worst outcome in pregnancy?

— Mitral regurgitation

— Mitral stenosis

— Aortic stenosis

— Aortic Regurgitation

$ Patient give birth to Rh positive baby, she took anti-D immune globulin during pregnancy, how
much will give her after birth

— 300

— 1000

— 2000

— No need

$ endometrial hyperplasia with atypia unfit for surgery what’s your management

— Chemo

— Radio

— continued progesterone

$ Female pregnant with recurrent late deceleration and fetal bradycardia and dilated 6cm

what to do ?

— Emergency CS

— Reassure

— Induction

A lady in labor and the fetus in severe distress FHR 70, Station -2, PG given, fully dilated cervix, good
contractions, what will u do?

A- ventouse

B- observe

C- reassure

D- encourage to push ✅

Lady presenting with history of 2 preterm labor, And currently presenting with minimal vaginal
bleeding / spotting, what is the next step:

A- estrogen

B- progesterone ✅

C- LH

$ Pregnant woman at 12 week gestation known to have seizure disorder

on phenytoin (last seizure attack 6 years ago), what to do about her

medication?

— switch the medication

— continue the medication

— decrease the dose

— increase the dose

$ Pregnant 15 GA Known case of epilepsy on phenytoin last attack 6

years ago came to antenatal visit ?

— stop phenytoin

— change it to safe medication

— refer her to neurologist to stop medication

— continue the same

Female patient presented with menorrhagia and a large fibroid. The doctor decided hysterectomy but
she declined as she want to preserve fertility. What is the most appropriate alternative management?

A. NSAIDs.

B. Oral progesterone.

C. Uterine artery embolization.

(Myomectomy was not among the choices)

$ Endometrial ca risk factor:

— Age

— PCOS

— HTN

— MUltiparity

$ A pregnant lady presented to the clinic, everything was normal

except she was having Barton hicks and rubella antibodies were

negative. What is your next step?

— 1. Follow up after 1 week

— 2. Do rubella antibody titer

— 3. Do glucose tolerance test

— 4. Give Rubella vaccine now

$ 330- Which of the following occur with OCP?

— Decrease the risk of ovarian cancer

— Increase the risk of breast cancer

— Decrease endometrial cancer

— Increase risk of ectopic pregnancy

c/P of adenomysis what is definitive treatment?

hysterectomy ✅

nsaid

40 years mf co rt lower fossa pain and vomitting for 2 day

pt alert vital stable

abd soft lax tender in it lower fosa pain and + rebound tenderness

CT Findings:

The right ovary is enlarged measuring 5 x6 cm with decreased enhancement associated

With large right ovarian cyst measuring 5x 5 cm. The left ovary measures 2.7 x 1.6 cm

associated with large cyst measuring 7x 6 cm. Minimal pelvic free fluid

— L OVARIAN TORSION

— OVARIAN CYST RUBTURE

$ Q mono mono twins cleavage

— 8-13 days

— <72 hr

— 4-7 days

Dichorionic-diamniotic :

Within the first 3 days after conception

Monochorionic-diamniotic:

Day 4–7 after conception

Monochorionic-monoamniotic:

Day 8–11 after conception

Monochorionic-monoamniotic (conjoined twins):

From day 12 after conception onwards

$ 30 w lady PPROM , no contraction, Next ?

— A- dexamethasone + erythromycin +ampicillin

— B- dexa + nifedipine

$ Scenario about placenta previa ( diagnosis is given) then asked:

what is risk factor for this condition?

— HTN

— DM

— Multiple gestation

$ Old, post meno bleed, had 5cm fibroid in the past now having the same findings on us  no
chanege, what next?

— Hysterectomy

— Observation

— Endometrial biopsy

— Myomectomy

$ Old age female [ 55 i think ] has lower abd. Mass, US showed large fibroid mass, which tumor
marker is correct ?

— Ca 125

— CEA

— Bhcg

— Ca 19-9

Chadwick's sign is one of several physical changes that occur during pregnancy.

It is an early sign that a person is probably pregnant. ✅

Note ' :

Blue cervix is called chadwick’s sign

$ preeclampsia with severe features, what are you going to give her

— clondine

— methyldopa

— hydralize

— sodium something

$ Which of the following indicate bleeding from a uterine source?

— Blood color

— Scanty

— Heavy bleeding

Patient presented with chronic pelvic pain and profuse bleeding Imaging showed: diffuse myometrial
thickness. What is the definitive treatment?

hysterectomy ✅

' Adenomyosis Definitive treatment is hysterectomy.

$ Female pt has a prev hx of gential warts, she did cryotherapy 2y ago. Now came for post coital
sca nty bleeding, what is the source of bleeding?

— Vaginal

— Vulva

— Uterus

— Cervix

primi came for antenatal visitis she twice had small for gestational age on abdomianl examination
which indicated IUGR. Which has the most highest diagnostic valvue?

— Non stress test

— Fetal kick chart

— Serial ultrasounds

— CTG

Pregnant lady with a previous DVT, What to give?

— Aspirin

— Enoxaparin

— Warfarin

— No anticoagulant

$ Pregnant with seizure given 6 mg sulphate then decrease to


4mg. On Examination there is absent deep tendon reflex, what
to do?

Anonymous Poll

— Reassurance

— Increase Mg dose

— Stop Mg infusion

— Give ca gluconate

What considered the Best score on biophysical profile?

A- 8/10 ✅

B- 9/10

C- 12/1

$ 33 years old women with heavy menstruation. During examination noticed firm fundal mass. Next
in management?

— CT

— US

— MRI

— Biopsy

$ Pregnant pt at 18 wks, she has a hx of recurrent fetal loss. Now came to ER due to sudden fetal
parts expulsion What is the dx?

— Bicornuate Uterus

— Cervial incompetence

A scenario of patient came to clinic with preeclampsia without severe features ( I think at 32 or 33
weeks). What you do?

A- Admission for observation

B- Give appointment at outpatient clinic after 1 week✅

A pregnant patient GA around 37 or 38 weeks came with labour , with contractions and cervix dilated
and effaced.

A picture of urinalysis showing positive ketones and Proteinuria and glucose.

$ No vitals mentioned, the did not mention if she has gestational diabetes or not.

What is the most appropriate management?

— A- Induction of labor

— B-Discharge and follow as outpatient

scenario about pregnant female with hyperthyroidism on methimazole 15mg and still complaining of
symptoms. What will you do for her?

A- Increase the dose of methimazole to 20mg

B- Switch to PTU

Choose ( ️ if “ 2nd, 3rd trimester”

Choose * ️ if “ 1st trimester”

$ A female didn't remember her LMP. How to know the the gastrointestinal age?

— B- Abdominal U/S at 18 weeks

— Tranvaginal U/S as early as possible

— A- Physical Examination

$ female prenatal visit history of hypothyroidism on Levothyroxine , it's controlled and T4 tsh level
are normal , no mention in Q if she is pregnant or not , just prenatal visit A- increase dose

— Increase dose

— Continue same dose

— Decrease dose

$ Pregnant lady has hypothyroidism, how much do you have to increase her dose?

— 10%

— 20%

— 30%

— 40%

$ 20-Female came from contraception. She has a hx of endometritis and 4y ago she got pregnant
while the IUD was inserted. What is the absolute Contra indication for IUD?

— A -previous hx of PID

— B- Previous hx of pregnant while IUD was inserted

— C- abnormal gental bleeding

Mother 34 weeks (o+)came with spott bleeding, other things are normaland cervics closed?

A. Give anti d

B. F/up after 2 weeks

$ 30 week pregnant with 2nd degree vaginal prolapse,


What to do

— A- admit for observation

— B- urgent delivery with vaginal repair

— C- urgent delivery

Woman delivered a baby 10 min ago and the placenta is still there and had PPH

what is the type of PPH

Anonymous Poll

— Primary

— Secondary

— Tertiary

cord prolapse what do you see in the ctg:

A- prolong decelate+bradycardia

B- accelarate

C- bradycardia ✅

A doctor is ordering 75 OGT for a pregnant patient. Why did he order it?

A- History of GDM In previous pregnancy

30 week GA pregnant woman had pprom 3 days ago, now presented with fever and abdominal pain,
what is the mangement?

A- gentamicin, ampicillin, deliver ✅

B- gentamicin, ampicillin, observe

C- observe

D- find the cause of fever and treat accordingly

————————

+ PPROM which is managed by:

- 24-33 6/7 weeks of gestation

• Expectant management

Expectant management:

! A course of betamethasone

! GBS prophylaxis

! Tocolytic drugs (Should not be administered for more than 48 hours. Or to patients who are in
advanced labor (>4 cm dilation) or who have any findings suggestive of subclinical or overt
chorioamnionitis.)

! Magnesium sulfate (if preterm delivery <32 weeks is anticipated” at risk of imminent delivery”)

• Prompt delivery in:

! Patients with signs of intrauterine infection, abruptio placentae, nonreassuring fetal testing, or a
high risk of cord prolapse is present or suspected

——————

$ What would see in 15 weeks of gestation:

— 20% increase in plasma volume

— increase in plasma volum 40%

— Increse in hct 20%

— Decrease in hct 20%

$ 5. 34 or 37 weeks with active labor , 4cm dilated, intact membrane. No vaginal bleeding.
Abdominal pain is increasing in frequency. Uterus is tender. - all findings are just going with active
labor- Next step?

— CBC

— US

— CTG

$ 37 weeks in active labor dialted 4cm intact membrane. CTG is normal except for recurrent
variable deceleration. Next step?

— CS

— ROM

— Observation

— Give tocolytic

Female had history of preterm labor at 34 wks and now she is on 24 wks what is the highest
diagnostic value for her case:

A. Cervical length measurement ✅

B. Speculum

A 27 year old pregnant lady, GA 39 came with a gush of watery fluid.

On speculum exam there were pooling of fluids in the vaginal vault. A Nitrazine test shows a PH of 7.

Which of the following is another highly diagnostic test for this condition?

A- indigo carmine injection in to the amniotic sac

B- Methylene blue injection in to the amniotic sac

C- US to assess the amount of amniotic fluids

D- Fern test ✅

How to confirm the fluid is amniotic fluid

(Speculum was done in Q)

A- Litmus test or methyl not sure ✅

$ 30 wk pregnant came for check up , Have mid systolic murmur radiate to carotid in left eternal
border with no diastolic sound

— physiological murmur of pregnancy

— AS

— MS

$ Pregnant with reduced fetal movements what is your next step?

— CTG

— Biophysical profile

— US

— Flow chart to count fetal movement

A pregnant lady did OGTT and they give the result ( only one of the 3 readings were abnormal ) what
is your management:

A- repeat OGTT

B- diet

C- Order fasting blood glucose

D- order HgA1C

$ Pregnant in labour, she takes heparin, post delivery she has heavy bleeding? What to give?

— A) FFP

— B) Portamine sulphate

— C) Vit K

$ A patient with post-partum hemorrhage who was resuscitated then they found that there is
persistent bleeding at several puncture sites what is the next important step?

— A) Oxytocin

— B) Reversal of coagulopathy

— C) Prostaglandin

+ She is bleeding from the puncture sites -> suspecting DIC -> FFP and consider cryoprecipitate

When to do a full congenital anomalies US?

A. 14-16 weeks

B. 18-20 weeks✅

C. 22-24 weeks

D. 24-26 weeks

Pregnant lady did CTG and showed fetal abnormalities ( no other details)

What will do INITIALLY?

A- oxygen

B - IV fluid

C- lateral position ✅

D - C/s

$ Female pregnant at 33GA, presents with multiple lumps around the areola, soft superficial
uniformed in size, limited to the areola. Most likely diagnosis?

— Montgomery’s Tubercles

— Breast cyst

— Lactiferous ducts

— Mondor's disease

$ pregnant woman present to the ER with moderate vaginal bleeding. O/E: gaurding, rebound
tenderness and shoulder pain (ectopic pregnancy). what is the Mx?

(no labratomy among the choices i’m sure 100%)

— A- methotrexate

— B- laparoscopic

$ What is the most common complication of GDM?

— Congenital anomalies

— Macrosomia

— Shoulder dystocia

— Hypoglycemia

$ 1227- Female with polyhydramnios you will console her about potential associated with

— Trisomy 21

— Renal agenesis

— Uterine rupture

— Fetal restriction

$ Female with breech baby will do C/S when to give the antibiotics?

— preoperative order

— after closure the wound

— before umbilical clipping

CTG with prolonged deceleration. What to do?

A) emergency CS

B) hydration, reposition and assess in 20 mints ✅

$ years old lady lost her pregnancy at 7 weeks GS , she is heavy smoker 20 s, per day she did not
take any supplements nor folic or iron for her last pregnancy, what is the couse of her Abortion?

— Family history

— Smoking a20 s per day

— No risk factor

— Folate acid def.

— Iron def

$ female in her 20s came to antenatal clinic, us shows polyhydromiuns. most prominent risk factor
for polyhydramnios

— A- DM

— B- Chromosomal abnormalities

— C- duodenal atresia

$ pregnant lady 16 weeks presented with calf pain and erythema Hofmann test positive what is the
most appropriate to reach a Dx?

— d dimer

— compressive US

bishop score , 3 cm dilated -2 station posterior presentation cervix firm , 80% effacement:

1- ripening agent ✅ ✅

2- IV oxytocin

3- artificial RO

3 cm > 2 point

-2 station >1 point

Posterior presentation > 0

Cervix firm > 0

80% effacement > 3

So total score is 6 ✅

Full recall

41 weeks +5 days came for induction of labor. The patient had 2 previous uneventful CS or vaginal
delivery (i forgot which one)

bishop score:

position: anterior

Consistency: intermediate

effacement: 50%

Dilation: 2 cm

station: -2

which method is the most appropriate:

A- artificial amniotomy

B- an infusion of Oxytocin

C- Insertion of a balloon catheter

D- prostaglandin repining

I disagree with earth and my answer is B based on this

Pregnant came with candida like picture and then asked what’s the TTT ?

A-oral metronidazole

B-azithromycin

C-topical metronidazole✅

$ A pregnant lady at 37 weeks of gestation presented with rupture of membranes and she is
known to have recurrent HSV vaginal infections , what is your next step ?

— 1. Do c section

— 2 . Prepare for instrumental delivery

— 3. Do vaginal examination

— 4. Give acyclovir

Case of pregnant lady in labor in cervical examination there palpable orbital edge , nose , mouth ,
and chin what is fetal attitude of this presenation ?

- Occipital

- face ✅

- brow

- I forget

Infertile couple trying to conceive for 12 months with regukar unprotected ‘ her husband is healthy
intercource what important or most appropriate (not sure and i think with regular cycle)

Hysteosalpingogram

Semen analysis✅

A 12 cm ovarian cyst was seen in a patient during Cesarean section , what to do?

A- Cystectomy✅

B- No intervention

C- squeeze and rupture ( literally was written like that)

42 female, last 15 months heavy menstrual bleeding its progress, uterus is normal and ovary not
palpable ,us 3-2 lesion

A- adenomyosis

B- submucous leiomyoma✅

C- DUB

$ 30weeks Pregnant with pruritus and jaundice, Medically free, LFT: high ALT(124)+ AST (100)
very high ALP(700). Ask about diagnosis: , normal platelets, no mention for BP in Q. Likely diagnosis?

— Cholestasis of pregnancy

— Hepatitis

— Budd chiari

— HELLP syndrome

⏺ ️pregnant 12week have cervical lesion what to do

1-pap smear

2-cone biopsy

3-colposcopy ✅

23-years-old female asymptomatic incidentally finding of 5 cm simple ovarian cyst. Patient is normal
and vitally stable. What will you do?

A- Surgery (not sure of this option)

B- Cystectomy

C- Examine her after 2 weeks

The right answer should be no need for follow up ✅

$ Pregnant with hx of previous difficult Cs admitted for elective Cs What to do for prevention
adhesion?

— Maintain hemostasis

— Insert drain

— Cs before got in labour

— Abx pre op

$ Mother recently delivered her baby, the doctor encouraged her for breast feeding What is the
most significant response? and baby

— A) decrease centrocranial infection

— B) reduce primary immunity disorder

— C) increase bounding between mother

— D) reduce the hemorrhagic disease for newborns

$ Ways to minimize adhesions.

— Avoid gloves with talc

— Maintain hemostasis

— Proper handling of abdominal tissues

— Saline irrigation of the abdominal cavity

35 female Secondary ammenorrhea for 8 month next?

A- Pregnancy test

B- Vagianal us

C- Full history taking ✅

D- Physical exam

Mother 2 monthes post delivery .asymptomatic. culture screening of urine more than100000 Ecoli
sensitive to ciprofloxacin_nitrofurantoin anf trimethoprime sulphamethoxazole Ask about treatment:

A. ciprofloxacin

B. nitrofurantoin

C. trimethoprim sulph...

D. no need ✅

The baby heart auscultation on the mother umbilicus ,,which is the presentation?

A- sholder ✅

B- face

C- breech

Pregnant weeks 38 presented with bleeding she is known case of placenta priva When we have to
admit?

A.Immediate admission✅

B.Admission after CS planning

32 yo women has menorragia for few months had serous fibroid the size was 4x4

Appropiate management

-NSAIDs

-Hysteroctomy

-Myomectomt ✅

-uterine artery ligation

Bilateral ovarian mass with ascites, pleural effusion, CA 125 was 100, no description of the masses
themselves w mdri shu, then asks what is most important for diagnosing?

A- Tumor marker

B- Tumor histopathology ✅

C- MRI of soft tissues

D- Pelvic U/S

47 yrs female , every 2 weeks she has polymenorrhea BMI 37 ,most important ?

A- pelvic mri

B- U/S

C- endometrial biopsy ✅

D&C after this amenorrhea FSH normal:

A- Asherman ✅

B- Sheehan

C-Kalman

How to confirm the fluid is amniotic fluid (Speculum was done in Q) :

A- Litmus test or methyl not sure ✅

B) Fetal hormones

$ Late deceleration pic and asked about the cause?

Anonymous Poll

— A- Uteroplacental insufficiency

— B- Head compression

__________________________

- CTG

. Early = head

$ Variable = cord

/ Late = placenta

__________________________

Pregnant in the 2nd trimester, she complained that, in this pregnancy, her abdomen larger than her
previous one. US finding: Twins. Which of the following is the correct statement regarding dizygotic
twins in correlation chorionicity and amnionicity?

A. Dichorionic and Diamniotic regardless of the sex✅

B. Monochorionic and diamniotic of the same sex

C. Conjoined twins with monochorionic monoamniotic

In a patient post mastectomy, they do for for her reconstruction from rectus

muscle, what is the vessels to be injured or effected? NEW20

A. Inferior Epigastric artery ✅

B. Superior Epigastric artery

C. Intercostal artery

D. Internal thoracic artery

$ 19 yrs old dysmenorrhea resolve on 3rd day and resolve after few , Sxs associated with sever
pain radiated to upper thigh, she had this for several years and getting worse

— Primary

— Premenstrual

— secondary dysmenorrhea

— endometriosis

Lady in labor with positive GBS what to give?

Abx ✅

ampicillin/penicillin

Vegan mom breast feeding most likely her mild will be deficient with what?

B12✅

Pregnant of twins, one has increased nuchal translucency in Ultrasound. What will he have?

A. Congenital cardiac malformation ✅ ( all chromosomal syndromes have cardiac disease)

B. Turner syndrome

C. Neural tube defect

Pregnant patient in 3rd trimester(was written like this with no GA weeks) with UTI (not giving labor),
what to give?

A. Nitrofurantoin
‫ يوم‬٣٠ ‫هذا برضو صح بس نوقفه قبل الوالده ب‬
B. Gentamicin

C. Ampicillin✅

27 weeks GA , scanty vaginal bleeding.  Us showed placenta previa totals. She's hemodynamiclly
stable. Ctg normal. Appropriate management =

A-Oxygen

B-Steroid ✅

C-Tocolytic

D-ABC

Most common cause of miscarriage in first trimester

Chromosomal abnormalities ✅

Obese pt with irregular menstral cycle

Biopsy show hyperaplasia

What will be cause?

Adrenal hyperplasia

Hepatic dysfunction

Gentic matution in endometrum

Converstion of estrogen✅

Physiological changes in pregnancy that can lead to heart failure in patient with mitral stenosis?

A- Increase minute ventilation

B- Increase RBC mass

C- Increase renal plasma flow

D- Increase plasma volume✅

25 years old primigravida 20 weeks GA history of mitral stenosis due to rheumatic fever what
physiological change makes her at high risk for heart failure?

A- increase RBC mass

B- increase stroke volume ✅

C- increase minute ventilation

D- increase Renal plasma volume

A pregnant lady complains of vaginal bleeding, shes a confirmed case of low lying

placenta, no

active bleeding now and no pain. Next step in management?

A- Ultrasound

B- Biophysical profile

C- CTG✅

D- Emergency delivery

pregnant 31 weeks came with uterine contraction and cervix closed what to give

A- Nifedipine

B- Terbutaline

C- Indomethacin

D- Corticoseroid ✅

$ Female 27 years old, she is asymptomatic, her last pap smear was 3 years ago and it showed
unconcerned

squamous cells. What is the most appropriate thing to do?

Anonymous Poll

— A-Repeat pap with cytology

— B-No need and reassure

— C-Colposcopy

27 yrs old female did pap smear 3 years ago and was normal what to do

1. No further testing

2. Colposcopy

3. Repaet pap with cytology ✅

$ Female used forcebs in delivery results in laceration in cervix and vagaina and pph, what’s the
reason for pph

Anonymous Poll

— Atony

— Retain placenta

— Genital track trauma

reduce IUGR and preterm ?

A-aspirn ✅

B-prostaglandin

$ 34 weeks presented with vaginal bleeding more heavier than her menses with regular utrine
contraction 3 in 10 min Baby: transverse lie back of the baby down, the placenta posterior and fundal
Membranes: bulging from the vagina Cervix: 3 Cm What to do:

— CS

— ECV

— Reassurance

— Tocolytic

$ female 7 week gestation with vaginal spotting, otherwise normal, what to do ?

— A-evacuate

— B-reassure

$ picture of peripheral blood smear with target cell and hyper-segmented neutrophil “

in scenario pregnant lady with fatigue and pallor, laboratory evaluation showing macrocytic
hyperchromic RBC, she had deficiency in which of the following

— Iron

— Vitamin B12

— Folic acid

picture of laparoscopy showing endometriosis “ in scenario lady with dysmenorrhea and she use
NSAID with no improvement, which one most appropriate drug for her:

oral contraceptives pills 0

Pregnant with prolonged ROM she’s planned for CS when to give ABx:

A- in pre operation preparation ✅

B- after OR

C- after cord clamping

A 37 year old woman comes to you with infertility. Semen analysis of the husband is normal and the
woman has normal, regular cycles and normal FSH and LH levels. However, US revealed many
fibroids disrupting the uterine cavity. What is the best way to restore her fertility?

a.Surgical removal of the fibroid ✅

b.Reassurance

c.GnRH to help shrink the fibroid

$ what is the commonest type of degenerative of fibroid in pregnancy?

— Red

— Cystic

— Fatty

— Hyaline

$ What happens to leiomyoma during pregnancy?

— a) Rapid growth

— b) atrophy

— c) calcification

97 answered https://t.me/obgynq/497 https://t.me/obgynq/498 OB-gyn, [Nov 9, 2022 at 3:19 AM]

45 years old male complaining of rash in hand and sole with history of 2 weeks painless ulcer in
genetelia and non-marriagesexual intercaurse .. Diagnosis?

a. Treponema pallidum ✅

b. Chlamydia

c. Hsv

d. hpv

Case of severe menorragea in 40s old female found to have uterine mass in US ( The scenrio was
litteraly like that ) , Dx :

A- parasitic fibroid

B-subserosal fibroid

C-submucosal fibroid

Answer: not clear question:

B’ unlikely to find in US

Mostly C

submucosal is the most common type causing menorrhagia

Primigravid come with labor for 4h Dilated 5 cm, effaced 80%, station +1 after 5h

there is no change in cervix, and contraction occur every 3 min. and stay for 60 sec. What

to do?

A. Instrument use

B. C/S

C. IV oxytocin

D. Wait for 2h✅

Couple want to conceive , male athlete and fertile , female high bmi

A) infertility test for f

B) diet for f

C) diet for f and infertility test for both✅

Pregnant with herpes treated with Acyclovir. Rationale of acyclovir usage?

A. Increase immunity of the mother

B. Decrease placental transmission

C. Decrease herpes simplex virus infection

D. Decrease shedding of the virus and duration

39 year old female presented with menorrhagia , US uterine examination revealed fibroid ,, where is it
located

⁃ submural

⁃ Intramural

⁃ Subserosal

-Submucosal fibroid✅

40 weeks gestation G3 p2 presented in labour ruptured membrane 5cm cervical dilatation for 3h 60
% effaced head station -1 Baby non stress test reassuring mother good vitals

What is the appropriate plan of management?

A) urgent c/s

B) observation✅

C) augmentation of labour by oxytocin

D) other non relevant

Spontaneous misscarriage percentage:

10-15% ✅

Dauring labour (I think in 2 stage) the midwife suggested a mediolateral episiotomy.

What is the advantage of mediolateral epesiotomy:

Easy repair

Less infection

Tighter introites

Less anus extensions✅

$ Pregnant in third trimester presented with bleeding, cervix was 7cm dilated.

Whats the type of bleeding?

Not mentioned if there was contraction

— post partum

— antepartum

— intrapartum

Pregnant female presented to the clinic with giant vulvar wart, how would you treat it?

A. Cryotherapy ✅

B. Electrocautery

C. Observation

Pregnant with hx of previous DVT , what to do?

A- Warfarin

B- LMWH ✅

C- No need

Woman comes in for c-section. You give her a pudendal


nerve block. All nerves around are sensitised except one:

A.Rectum ✅

B.Perennial body

$ years old female asymptomatic. On examination, there's grade 3 posterior vaginal wall prolapse.
she denies any symptoms. Which of the following is the most appropriate management?

— posterior colporrhaphy

— Conservative mx and observation

— surgical

$ GA38, positive for genital chlamydia, what most common infection in regard to the baby

— Eye ✅

— Ear

— Lung

Female patient with dysmenorrhea and u suspect endometriosis, what is the (most) appropriate step?

A)US — initial

B)diagnostic laparoscope — Best

C)CT

Patient with gestational DM, initial treatment?

A) metformin

B) insulin I.V

C) insulin S/C

D) Diet and lifestyle modification ✅

$ signs of preeclampsia with severe features in 33 GA , CTG normal , what will u do

— A- bed rest till 37 wks and revaluate

— B- bed rest till 34 wks and IOL

— C- admission and induction of labor

— D- admission and CS

__________________________________________________________________________________________

whitish cheesy vaginal discharge what is ur mx ?

A- topical azole ✅

1 + 2 = Topical azole

3 ‍♀ ️+ 2 = oral

__________________________________________________________________________________________

Pregnant came to the ER today with moderate bleeding , what next step? (Not mentioned if there’s
Hx of placenta previa) full recall

A. Transfer to US✅

B. Insert 2 large IV cannula

C. Emergency delivery

D. Pelvic examination

34 pregnant women come to ER complinig of vomating and abdoinal pain and foud to have HTN

A deliver the pf

b stabilization pt give mg and wait till 37

c stabilization and give abx]

26-year-old female came to the Clinic complaining of severe lower abdominal pain tat started just
before her menses and disappeared on the 3rd day, the pain so severe that affect her daily activity
and prevent her from going to work for many days. Her boss is not happy. Which of the following is
the most appropriate management?

A- hysterectomy

B- Paracetamol

C- OCP ✅

No NSAID in the options

40 y.o female with past medical history only with dysmenorrhea. Now she presenting with severe
dysmenorrhea ( not bad recall!)?

A-Ovarian thecoma

B-Ovarian fibroma

C-Endometriosis ✅

D-Leiomyoma

$ 34- Female ( i think post menopausal) came with mass on the labia red and bleeds on touch (I’m
not sure of the scenario) + Pic

Anonymous Poll

— 1-carbuncle

— 2- furuncle

— 3-caruncle

____________________________________________________________________________________

6 Pelvic inflammatory disease 6

Acute salpingitis case not responding to ceftriaxone treatment what is the cause ?

A-Adenovirus

B-HSV

C-Gonnorhra

D-Chlamydia ✅

A patient with PID (brown discharge), she was treated with IV ceftriaxone but did not improve. What
is the most likely causative organism?

A- HSV

B- Neisseria gonorrhoeae

C- Chlamydia trachomatis ✅

PID treated by ceftriaxone?

A. Neisseria ✅

B. chlamydia

C.herpes

https://t.me/obgynq/527 OB-gyn, [Nov 10, 2022 at 3:15 AM]

__________________________________________________________________________________________

7 PID treatment #

N. Gonorrhea Ceftriaxone

chlamydia Doxycycline /Azithromycin

7 PID pathogen 7

No respone to Ceftriaxone ? Pathogen is chlamydia

No respone to Doxycycline /Azithromycin? Pathogen is N. Gonorhea

__________________________________________________________________________________________

Young female with irregular cycles , 1st menarche since age of 14 yrs , high BMI , she started to gain
wt since menarche , also there is acne , facial hirsutism , lab show high testosterone level Your dx

A) premature ovarian failure

B) PCO syndrome ✅

Female with breast tenderness , severe depression inability socalize , anixiety , crying the signs
affecting life and work started 1 week before menses disappear in 3 day

A. Transdermal estrogen patch

B. SSRIs ✅

C. OCP

Young female not pregnant with hx of PE 1 year ago, presented with a swollen leg. US showed
proximal DVT, most appropriate management:

A- Long term anticoagulant therapy ✅

B- Anticoagulant for 2 weeks

C- Thrombolysis

D- IVC

Female with polyhydramnios you will console her about potential associated with

A- Down syndrome ✅

B- Renal agenesis

C- Uterine rupture

D- Fetal restriction

32 old came to gynecology clinic for her dysmenorrhea.

Which of the following is history evaluation is importent?

A. Medical

B. Surgical

C. Family

D. Menstrual ✅

Case of molar pregnancy (very high Bhcg…), what is the highest diagnostic test??

A. US

B. Direct biopsy

The correct answer should be “ histopathology “ 8 9

Multiple uterine fibroids Failed ivf 2 time :

- Myomectomy

- Gnrh agonist

Drug given to reduce the size of fibroids

A- GNRH ✅

B- Contraceptive pills

A 37 year old woman comes to you with infertility. Semen analysis of the husband is

normal and the woman has normal, regular cycles and normal FSH and LH levels.

However, US revealed many fibroids disrupting the uterine cavity. What is the best way

to restore her fertility?

a. Surgical removal of the fibroid

b. Reassurance

c. GnRH to help shrink the fibroid

Before fibroid operation, to decrease fibroids size :

A. GNRH ✅

B. Progesterone

C. OCP

D. Steroids

Female with history of previous abortion , she is having multiple uterine fibroid. Which of the following
will help her the most to get pregnant

A- Myomectomy

B- Uterine artery embolization

C- Medical therapy

D- GNRH to decrease the size of fibroid

Multiple uterine fibroid desire pregnancy?

- Myomectomy

- Uterine artery immobilization

- GnRH to decrease fibroids size

28 years old female tried to get pregnant for many years, did IVF 2 times, now she’s pregnant in i
forgot which week but was 2nd trimester and US showed multiple fibroids, how to treat her without
harming her pregnancy?

A- myomectomy

B- GnRH

30y old female complain of post coital bleeding and irregular cycle for 3 months, what is the next
step ?

A- pap smear

B- pelvic US

Case of hyperemesis gravidaum What to find to dx in urine :

A- Ketone ✅

Typical picture of complete mole fundal heigh > GA :

Suction& evacuation ✅

Dilation and curretage

Methotrexate

__________________________________________________________________________________________

Passage of vesicles:

Complete mole ✅

Partial mole Incomplete abortion

Lady 29 week pregnant Came with bleeding and Vesicle Structure came out what is your Dx ?

- partial mole

- complete mole ✅

- threatened abortion

- missed abortion

Passage of Vesicles = complete mole

Partial mole = Fetal Parts

__________________________________________________________________________________________

37 weeks patient came with cord seen , ressureing ctg hr 120 , cervix 7 cm ,

1- fetal monittoring

2- ventose

3- forcepos

4-c.s ✅

post delivery 10 days or weeks came with fever and chills, there is swelling and tender with cracked
nipple

1- puerporal sepsis

2- breast abscess ✅

3- cardiomypathy

Female came in with abdominal cramps, long scenario, but HCG was 200 or 2000, ultrasound
revealed no embryo in the uterus?

ectopic pregnancy ✅

Female who is pregnant but doesn’t know her LMP how do we determine the gestational age?

(No crown rump length was mentioned in the question)

A- physical exam

B- TVUG ✅

Female, G1P0, missed her period for 3 months, has irregular period, doesn’t know gestational age.
On Ultrasound, 11 weeks was calculated. What is the most likely mthod of dating gestational age
here?

A-crown-rump length ✅

B-femur length (and head circumference?)

C-estrogen or progesterone? Level

D-abdominal circumference

Female 35 y/o primigravida, smoker,

came for antenatal visit, history of

subfertility, and family history of diabetes militus in second degree relative. What’s the biggest risk
factor for her? (i don’t know for what exactly but i guess for pregnancy complications)

A-Smoking

B-Subfertility

C-Age ✅

D-Family history of DM

Female 34 y/o primigravida, smoker, came for antenatal visit, history of

subfertility, and family history of diabetes militus in second degree relative. What’s the biggest risk
factor for her? (i don’t know for what exactly but i guess for pregnancy complications)

A-Smoking ✅

B-Subfertility

C-Age

D-Family history of DM

Both came one as 34 years old and one as 35 years old

All references I looked at suggest that age is strongest risk factor if the patient was 35 or more

But if he was 34 I would go for smoking

Female, vaginal delivery, removed placenta completely by tugging on umbilical cord, shortly after,
bleeding profusely, PPH management?

A-Tocolytics

B-Packing

C-Check uterine contractions ✅

Pregnant at 39 wks. now in labor during the delivery you noticed the amniotic fluid is mixed with Dark
black-green what is the cause of this color ?

• A fetal aspiration

• B fetal distress ✅

• C placenta abruption

• D preterm labou

14 y female complain of vaginal bleeding in interval of 3 weeks to 2 months, she has normal
development and normal secondary features, every thing normal What to do?

A- Reassure✅

B- OCP

C- Order FSH and prolactin

Patient presenting with similar pictures with painfull vualva enlarged non tender inguinal lymph nodes
in and the asked about what’s the diagnosis?

A-Syphilis

B- Chancroid ✅

Case of PMDD, affecting life severely

A. -SSRI

B. -Progesterone suppository

5 days Boy with single artery umbilical cord

A. Dm mother ✅

B. Africa race

C. Low fetal risk

D. Risk of fetal malformation 80%

986- Middle aged female patient with history of myomectomy. in operation note: they enter uterine
cavity during surgery. What is the most likely developed in future pregnancy?

A. Placenta previa

B. Placenta acreta

C. Placenta Increta

D. Placenta percreta

Ectopic pregnancy case initial BhCG 2900, given methotrexate, one week later BhCG 6000 what to
do:

A- repeat methotrexate

B- Diagnostic laparoscopy ✅

Indication for Repeat dose

1- BhCG Less than 5000 or equal

2- 4cm or less

Indications for Surgical

1- BhCG Above 5000

2- Sign of Rupture

3- Absolute CI of MTx

Elderly has menorrhagia ( ‫ )لوطتو ديزت اهلام لكو‬they give US pic & has 1.2 not sure around 1-3 cm
mass?

1) polyp ✅

2) submucosal leiomyoma

https://t.me/obgynq/563 OB-gyn, [Nov 10, 2022 at 7:49 PM]

Asking pregnant women with no remarkable history on pregnancy came on 34 -37 not sure

The test results was ketone bodies positive and protein I think

BP and everything was normal

The option was

- CS

- observation

- induction of L

Observation

Pt complain of profuse discharge and tender vaginal fornix

A. Acute salpingitis ✅

B. Acute cervicitis

C. Appendicitis

-female with suprapubic pain with purulent discharge . Vaginal Ex tenderness in fornix

A-Acute cervicitis

B-Acute salpingitis✅

C-Acute appendicitis

D-chronic appendicitis

Pregnant 38 wks have positive nitrat

A- Cephlaxin ✅

B- Doxycycline

C- Increase fluid

D- nitrofurantoin

$ Pregnant 34wks complain of headache, change in visual acuity and RUQ pain BP hight don’t
remember the number. What’s the management

Anonymous Poll

— stabilize the pt give Magnesium sulfate and delivery

— stabilize the pt and give steroid and deliver

29 votes https://t.me/obgynq/567 https://t.me/obgynq/568 OB-gyn, [Nov 11, 2022 at 12:10 AM]

21 sep

A 30-year-old women K/O severe acne in which she takes isotretinoin came to the clinic she tolds
you that she wants to conceive and asking about if her medication affect her pregnancy?

A- tell her that it’s save in pregnancy

B- she can conceive immediately once stop the medication

C- She should wait 3 months after stopping the medication then conceive

( 1 month is the right answer )

Full recall

A female patient on isotretinoin for Acne. wishes to conceive. What should the physician tell her
about the appropriate time?

A.She can conceive as soon as she stops the isotretinoin

B.She canconceive while taking isotretinoin

C.She can conceive after 3 moths off of isotretinoin

D. She can conceive after 1 month of stopping isotretinoin ✅

$ Case of pregnant woman in 1st trimmest, she had one prior pregnancy and was uneventful. The
Patient is in clinic following up, she is a SC Patient that has about 1-2 pain episodes a year. What is
the best management for this patient:

Anonymous Poll

— a) Give hydroxyurea

— b) Exchange transfusion

— c) Simple transfusion

— d) Close monitoring and frequent followup

Female had ectopic pregnancy two years ago and treated with methotrexate, now wants to conceive:
what will you advice this patient: ( I think in Wafa but in a simpler form)

A) She should do IVF

B) Start folic acid

C) Close follow up for early detection of fetus location✅

(it was something along these lines

>> in wafa it said early detection of pregnancy)

$ Pregnant in first trimester showing s/s of cholestasis, when to perfrom procedure:

Anonymous Poll

— Now

— 2nd trimmest

— 3rd trimmest

— After

10 weeks pregnant complaining of biliary colic for the past 5 weeks. What is the most appropriate
management

A - Laparoscopic cholecystectomy now

B - Laparoscopic cholecystectomy after delivery

C - Laparoscopic cholecystectomy in second trimester

D - Laparoscopic cholecystectomy in third trimester

$ Premature delivery of 23weeks old with fetal congenital anomalies. After delivery he needed
intubation and resuscitation. The mother who also works at the same hospital is refusing to
resuscitate

Anonymous Poll

— do you respect her wishes

— consult hospital ethical commute

— continues with the resuscitation

$ A pregnant woman in her 10 weeks with recurrent biliary colicky pain, when to the surgery?

Anonymous Poll

— A-Immediate cholecystectomy

— B-Lap chole after delivery

— C-Lap chole 2nd trimester

$ Pregnant lady K/O SLE controlled on HQ, mycophenolate motefil. Asking about what drugs
adjustment should be done?

Anonymous Poll

— stop Mycophenolate and give methotrexate

— stop Mycophenolate and give azathioprine

$ Pregnant may be at 36 wk with this picture what will do for her?

— A. Immediate delivery

— B. Reassure and follow after two hr

GA 39 with active labour , fetal tachycardia 180 early declaration which one is the dangerous?

A- GA

B- Active labour

C- fetal tachycardia ✅

D- early declarations

A 14 yrs old female with irregular cycles, obese high BMI , management?

A-OCP

B-star metformin

C-Advise to loose weight and regular exercise✅

Female patient had fibroid and menorrhagia what is the most likely the location of the fibroid?

Submucosal ✅

Multiple small breast masses bilateral get worse prior menses

A- Fibrocystic ✅

B- Fibroademoa

History of multiple still birth, aptt prolonged. Dx

A- Antiphospholipid ✅

B- Protein C deficiency

C- Protein S deficiency

$ Female had infertility for 6 years, complains of severe dysmenorrhea

— lyomyoma

— endometriosis

— adenomyosis

Help syndrome in 38w (you will diagnose it diagnosis not given)

A- emergent deivery ✅

Pt did Myomectomy enter endometrial cavity ( risk of placenta inecrreta )

A. -increase ✅

B. -dec

C. -uknown

Hx of asthma in labor with pph doctor did massage but didnt help she stil in bleeding Which of the
following contraindications is the most appropite managment to her at this stage ?

A- oxytocin

B-misoprstol

C-methylergonovine

D- Carboprost ✅

30 yo primigravida, 30 GA diagnosed with preeclampsia What’s antihypertensive drug is CI?

A. Nifedipine

B. Captoppril ✅

C. Methyldopa

D. Hydralazine

Elderly female complaining of white cheezy secretions with itching , what is the diagnosis ?

A- diabetes millitus ✅

B- HTN

- female k/c of severe depression, Hx of suicidal attempts, she is controlled on

paroxetine, now she is pregnant ( in other scenario she is planning to be pregnant) what is

the best action:


A- stop paroxetine because of fetal malformation


B- continue paroxetine and control her depression


C- switch to other antidepressants


; be careful to the given Hx:


- if currently she is pregnant: as she is controlled now, continue on paroxetine ( or any other

antidepressant she is controlled on it) and educate her about the possible side effects to the

baby which is less in rate than having maternal relapse

( don’t break the boat )

- if she is planning to be pregnant and still have time, you can switch to another

antidepressant safe for both with close follow up and assessment.

- if untreated before, Sertaline is a good choice to start with and to be pregnant on it.

< corrections and explanation will be added to the booklet

! Pregnant woman was on oral iron for 12weeks Still her labs IDA what you will do ??
— A-Continue oral
— B-Iv iron
— C-Blood transfusion

B

Qs ! Pregnant came c/o N V and right abdominal pain obstetrician excluded pregnancy related
conditions what is the most likely Dx
— 1. Gastritis
— 2. Pancreatitis
— 3. Cholecystitis
— 4. Appendicitis

D

! pregnant 31 weeks came with uterine contraction and cervix closed what to give

— A- Nifedipine
— B- Terbutaline
— C- Indomethacin
— D- Corticoseroid

D

! Praimigravida 32W with type 1 DM presented with abdominal pain and uterine contractions her os

is closed what to do?


‫ اﺳﺗﻔﺗﺎء ﺳري‬
— Steroid and increase insulin
— Steroid
— increase insulin and tocolytics

A

Female pt have history of previous myomectomy and several induction for fertility due to polycystic
ovary Now present with uterine bleeding what's the cause
1 endometrial hyperplasia
2 endometriosis
3 adenomyosis

3






Pregnant kc of idiopathic thrombocytopenic purpura developed PPH ?
— FFB
— RBC
— Platelet transfusion
— Cryoprecipitate

C

16 year old , amenorrhea for 2 months and galactorrhea for 3 months , what is the most important
investigation ?
— A-Prolactin
— B-Progesterone
— C-Oestrogen
— D-LH

A

How to defrinitaite between benign vs malignant ovarian tumor ?
A. Hypoechoic
B. Bilateral
C. Cyst with septate

Ans. A

Qs 20 years old pregnant known case of Idiopathic Thrombocytopenic purpura. Her delivery was
complicated by post partum hemorrhage. What will you give her next?

A. Packed RBCs
B. Fresh frozen plasma
C. Cryoprecipitate
D. Platelets transfusion

Ans. D

! Management of PROM in active herpes simplex virus

— CS
— Iv acyclovir

A

pregnant+ DM, complication ➡ Congenital malformation, RDS (after birth)
Pregnant+ preeclampsia complication➡ IUGR, oligohydramnios
Sickle cell anemia + pregnant complication ➡ IUGR
Sickle cell trait + pregnant complication ➡ UTI


Q about lesion in labia majora in post menopausal female showed dysplasia (carcinoma in situ I think)
what to do:
A. Steroid cream
B. Local excision
C. Vulvectomy
D. Repeat test after 6 months

B

- Pregnant came with labour for 2 hours presentation how to confirm diagnosis (ctg 3 contractions per
10 min+ cervical diltation 3 cm)
A-repeat vaginal examination after 2 hours
B-biophysical profile
C-continous ctg
D-fetal sample something like that

A

! Female regular period women want to conceive. What is the laboratory indicative of ovulation?

— Urine FSH LH
— Us
— Estrogen
— Progesterone21

D

Primary infertility with scanty irregular cycle and very high FSH, most important work up?
A-FSH/LH in urine I think
B-CBC
C-Pelvic US
D-chromosomal analysis

D

Qs 18 yr complain vaginal pruritus & discharge 5 days ago, no fevers or abd pain.PEx: normal genitalia.
speculum exam:thick yellow cervical discharge with easy bleed on touch.
Cervical swabs done & reveal no organism on gram stain. most appropriate TTT?
A Azithromycin and ceftriaxone
B. Ceftriaxone only
C. Azithromycin only
D. Metronidazole only

D
31w pregnant lady complains of small and slow vaginal bleeding, shes a confirmed case of low lying
placenta,
A-expectant management
B- Biophysical profile
C- CTG
D- Emergency delive

C

Patient primigravida 6 weeks presents with severe lower abdominal pain radiating to the shoulder.
vitals 90/50 , HR 118, US: No intrauterine pregnancy . What is the most appropriate next step
A-Laparoscopy
B-Methotrexate
C-Prostaglandin

A # Right answer should be LAPAROTOMY, As it’s a case of unstable ruptured ectopic pregnancy.

pregnant diagnosed placenta previa what is the most associated with it
A- post coital bleeding spotting
B- profuse bleeding at the first episode

A

primigravida patient presented in labor. O/E: the cervix is 5cm dilated and the fetus is in a station O
with cephalic presenting part and this state for 4 hours even the oxytocin had been taken. CTG shows
one variable deceleration. what is the management for this patient?
A stop oxytocin
B immediate CS
C follow up
D instrumental delivery

A

! CTG shows fetal tachycardia and prolonged PROM. Not in labor. What to do?

— A- Repeat cervical examination


— B- Give antipyretics and reasses after 2 hours
— C- CS

C

Qs ! Long scenario female pregnant of baby with down syndrome, how to diagnose?
— A- high inhibin, high HCG, low estriol, low AFP
— B- Low inhibin, high HCG, High estriol, low AFP

A


Highest causing factor of endometrial cancer:
A- late menarche.
B- early menopause.
C- DM.
D- progesterone tumor

C

Qs Pregnant 37 weeks gestation presented with moderately severe vaginal bleeding, was diagnosed
as placenta abruptio, Ultrasound revealed 32 weeks fetus. (I can’t really recall if cervix was 7 cm with
3 contractions per 10 min and fetal head is +1) What is your most appropriate next action?
A- induction of labor
B- C/S
C- fentose
D- forceps

A?

Female complaining of infertility normal labs and what is the diagnosis?
A- bilateral hydrosalpinx

Qs female want to conceive get to clinic she is SLE on Plaquenil what to do?
A: keep using it
B: stop it
C: refer to Rheumatologist to stop it

A
Calculate iron intake for pregnant lady 75 kg ?
A. 1mg
B. 1.8 mg
C. 3 mg

1
Qs Normal pregnant women at 12 weeks, Indication of severe preeclampsia :
A- Increase cr
B- Increase urea
C- Increase Na level
D- No platelets in choice

A

21 years old pregnant , 8 weeks GA came with bleeding but stable , US done showed non viable fetus
and age 5 weeks , appropriate mx ?
- Misoprostol
- Hysterotomy ( not hysterectomy )
- Hypertonic saline infusion

Ans. Expectant management?
Pregnant at 32 week of gestation with renal colic. She states that she has had 2 previous episodes of
the same presentation during this pregnancy. Investigation?
A- Renal US.
B- Cystoscope.
C- Cystogram.

A

Pregnant take oxytocin and then the fetus Develop tachycardia ?
A- Stop oxytocin ✅

postpartum patient she feels dyspneic whenever she’s lying down on the bed what is the possible
diagnosis:
A) Pulmonary embolism
B) Peripartum cardiomyopathy
C) Myocardial infarction

B

.30 something year old lady presented 1 week after birth with Bilateral crackles and dyspnea What is
the likely cause?
A- MI
B- Pericarditis
C- Peripartum Cardiomyopathy

C

27 years old women come to the ER complain of mild vaginal bleeding, LMP was before 9 w, Ex
revelead soft uterus felt above the symphysis pubic, what is the most appropriate next step?
- pelvic US to locat placenta
- pelvic MRI
- CT abdomen
- CT chest abdomen pelvic

A

Polyhydramnios cause?
A- Anencephaly
B- Postterm pregnancy
C- Maternal ingested NSAIDs
D- Posterior urethral valve

A




121- postmenopausal woman complaining of 1 year hx of recurrent vulvar itching associated
sometime with blood streak secretions , recently develop pea size mass in the labia?
a) bartholin cyst
b )cystic adenosis
c) bartholin gland cancer
d) squamous cell cancer of vulva

D

38-year-old women is seen for the evaluation of a swelling in her right vulva. She has also noted pain
in this area when walking and during coitus. On examination a mildly tender fluctuant mass was
noticed just outside the introits in the right vulva. What is the most likely diagnosis?
A) Bartholin’s abscess.


Qs Vulvar lesion at 5 o'clock, inflammatory changes up to cervix, red , edematus :
Qs Vulvar lesion at 7 o'clock :
A- Carbuncle
B- Bartholin
C- SCC
D- furuncle

B
% Bartholin’s abscess.

Bartholin’s glands are normally located located at the 5-o’clock and 7-o’clock position. When infected
(abscess), they are usually unilateral, tender, and are surrounded by edema and erythema.

Most important risk factor of abrupTIo placenta ?
A- Short umbilical cord
B- Hypertension
C- uterine fibroid

B

A female presented complaining of abdominal pain with watery and greenish vaginal discharge.
She recently entered IUD. What is the diagnosis
A- PID
B- Bacterial vaginosis
C- Uterine rupture

A

Pt come with abdominal pain no sac in uterus but with adnexal mass and long senario ,vital stable:
A-Stable ectopic pregnancy.
B-Rupture ectopic pregnancy.
C-Molar pregnancy

A
Why is ACEIs contraindicated in pregnancy?
A-It causes fetal Kidney teratogenicity
B-Sudden decrease of Bp

A
Other recall: Why ACIs is Contraindicated during pregnancy :
fetal anomaly
Renal impairment of the fetus

Differ acc to trimester
# First trimester: anomalies

# Second and third: renal impairment


“But in general, renal impairment is more established, so choose it”

40 weeks GA pregnant lady with 12cm ovarian cyst.
A-Observe
B-Cystectomy
C-Nonhorectomv

A?

Female with infertility since 6 years and have dysmenorrhea not responding to NSAID what is DDx?
A- Endometriosis
B-?

Pregnant lady at term presented to the DR with regular contractions and fully effaced and active
labor, during the delivery everything was normal except the baby head is +2station ,what is this next
step?
— CS
— Ventose
— Stop oxytocin

B

! 27 y old female present with acute lower abdominal pain radiate to left shoulder whats is the

highest diagnostic investigation


— Pelvic CT
— Abdominal MRI
— Pregnancy test

C

Snow storm appearnce , what is dx ?
A) complete molar hydatidiform
B) partial molar hydatidiform
A
about pregnant in 3 trimester how can we measure the weight of baby ?
A) CRL
B) Abdominal circumference
C) Femur length

B

patient with fibroid, what contraceptive to give?
A) condom
B) IUD
C) OCP
D) POP

C

Pregnant women with Twin, cbc shows IDA , which of the following indicate to give IV iron ?
A. Hb 10
B. Modrate to severe Anemia
C. Multiple pregnancy

How to calculate the Estimated Delivery Date?


A. Day + 7 / Month +9 / Year +1 or 0 (depending on the month)

34 y.o woman came to the clinic complaining of milk-like discharge from her breast for the past 4
weeks, she has not menstruated in 2 months. Physical examination is normal with exception of
galactorrhea, asking about what is the intial investigation:
A. Prolactin
B. Pregnancy test
C. Mammography
D. MRI

B

Qs. adnexal mass was felt in healthy female with normal pelvic examination, no pregnancy, had her
menstruation 2 weeks?
a) follicular cysts
b) luteal cysts
c) pco

A
Corpus luteal cysts in the normal menstrual cycle may have a variety of appearances on ultrasound.
They are unilocular and can contain internal debris (hemorrhage) and thick walls. They can also be
enlarged, up to 8 cm, but typically resolve spontaneously [7].
the corpus luteum may occasionally become enlarged and painful due to hemorrhage.
First 2weeks =follicular ,, Last 2 weeks =luteal
14 days: late follicular ,,, Early luteal
Qs Female with suprapubic tenderness and non purulent vaginal discharge Type of infection
A. Vulvar
B. Vaginal
C. Cervicitis
D. Uterocervicitis

D

Mother smoker whats the risk on fetus?
A. Low birth wt
B. Two vessels cord
C. 80% risk of malformation (not sure)
D. Reduce blood oxygen or increase co not sure

A

Long case of pregnant 34 w you did everything and it’s normal what to do
A- Admit for observation
B- See next week
C- Do glucose tolerance test
D- Deliver now

B
Williams Obstetrics: PRENATAL VISITS: Traditionally scheduled at 4-week intervals until 28 weeks, then
every 2 weeks until 36 weeks, and weekly thereafter.

Women with complicated pregnancies-for example, with twins or diabetes-often require return visits at
1- to 2-week intervals

Qs Preg female 39ws came as 1st visit with no complains Bp 100/80. HR 100 Very poor urine dipstick
pic attached showed Glu 4+ keton 4+ protien 3+ ph 5 What to do:
A. Urgent cs
B. IOL
C. Admit for observation
D. Discharge and see next week

B

38 wks gestation. Rupture of membrane for 24 hr. No confractions. Ctg reassuring. Management?
A. CS
B. IOL

B




Qs ( pic similar to this one ) the case was about a post menopausal women who didn’t complain of any
thing except a minimal bleeding when she touches the urethra (the radish spot), what is your Dx:
A.Urethral caruncle
B. Urethral prolapse
C. Vaginal atrophy
D. Furuncle

Woman, menopause, LMP a year ago, wt will change:
A. Testosterone
B. Estradiol
C. FSH

C

! Infertile women for three years came to clinic with her husband, she wants to conceive but she

couldn’t, semen analysis is normal, induction of ovulation by clomiphene citrate was done, (i think
every thing else is normal regarding labs) what to do next:
— IVF
— IUI
— laparoscopy
— induction again(with another drug not clomiphene)

A

A pregnant women with twins. 1st twin breach and 2nd twin cephalic. What will you do?
A-SVD
B-ECV then vaginal delivery
C- C/S

C

Qs Female presented with premature rupture of membrane with fever and vaginal discharge. The
uterus is tender to touch. What is the possible diagnosis?
A. chorioamnionitis.
B. UTI.

A

patient at 32 weeks GA presented to the obstetrics clinic with cervical dilatation station – 3
effacement 70% and mild abdominal pain and contractions What is the next most appropriate step?
A) Call neonatoligist, tocolytics, observe
B) Call neonatoligist, antibiotics, steroids
C) Call neonatoligist, steroids, deliver
D) Call neonatologist

C

patient PCOS what should be investigated
A- Glucose tolerance and lipid profile
B- Thyroid profile

A

Pregnant women 32 weeks complaining with vaginal bleeding, there is no history of contractions or
cervical dilation. What is the type of bleeding?
A- Early postpartum
B- Late postpartum
C- Antepartum
D- Intrapartum

C

female with placenta previa had severe bleeding. What is the most likely outcome post delivery?
a) Galactorrhea
b) Diabetes
c) Absence of menstrual cycle
d) Cushing syndrome

C ( Sheehan’s)

Patient with intermenstrual bleeding. What is the most appropriate investigation
A) CBC
B) TFT
C) B-HCG
D) US

D

14 years come with intermenstrual bleeding. What is the problem?
A- Metabolic
B- Endocrine
C- Genetic

B

Qs ttt of intermenstrual bleeding —> steroid ?.?.?.?


A 38 GA mother who who was an Rh positive baby and an Rh negative mother. What is the next step?
A) Amniocentesis
B) Emergent delivery
C) Reassurance

C

Mother devolved infection while she is pregnant, she developed an IgG What is the kind of immunity
the baby will have
A. Passive natural immunity
B. Passive artificial immunity
C. Active natural immunity
D. Active artificial immunity

A

23 wife medically free, regular cycle, c/o primary infertility for 3 months, Husband 26, medically free,
has 1 cousin autustic, 1 with trisomy 21, Most appropriate :
A- continue trying
B- semen count
C- gentic testing

A

35 y/o, primary infertility for 3 years, has 3 cycle of clomid as a case of polycyctic ovary, Husband
smoker.
A- semen analysis
B- husband prolactin
C- 2 day of cycle hormons
D- hystosalpingo

A

Married for 3 years, off contraceptive since 18 months and still didn’t get pregnant
A. IVF
B. Induction ovulation
C. Infertility investigation for both wife and husband
D. Continue trying

C

Case underwent salpingogram for infertility with occlusion in one tubes:
A. IVF
B. ICSI
C. Clomiphene citrate
D. UI-H

C

! lateral episiotomy better than middle why:

— A. Decrease the 4 th degree


— B. Better healing

A

Pregnant on antiepilptic drug what effect it on the baby ?
A. Congenital malformations

sudden severe lower abdomen pain in female with large collection cul de sac
A- rupture ectopic

Woman had 7 kids now she is complaining of a vaginal mass protruding more with cough what is the
next step
A- Vaginal speculum exam
B- Retrograde Cystourethrogram

A

Mother going for normal labour, which of the following can be worrying
A- Early deceleration
B- Fetal tachycardia 160 baseline and increased with contraction

B

Female c/o vag spotting ,preg at 7 wk, by TV u.s there is no intrauterine sac and no extrauterine sac
wts next:
A- Laproscopy
B- Repeat u.s and lab
C- Methotrex

B

Qs lady in labour and the fetus in severe distress FHR 70, Station -2, PG given, fully dilated cervix, good
contractions, what will u do?
A- ventose
B- observe
C- reassure
D- encourage to push

D
Mother give birth to a baby then got Rh injection, which pt needs this injection?
A- Mother Rh- , baby Rh+
B- Mother Rh+, baby Rh -

A
! 37 weeks in active labor dialted 4cm intact membrane.

CTG is normal except for recurrent variable deceleration. Next step?


— CS
— ROM
— Observe
— Give tocoloytics
B
! 2- 50 y/o female present with uterine spotting, no history of wt loss, most probable cause:
— Endometrial Hyperplasia
— Endometrial hypoplesia
— Uterine cancer

B

# Previa : multiple gestation

# Accreta : cs or endometrial damage

# Abrubtion: htn

# IUGR is with oligohydraminus, HTN / pre eclampsia

# Down = duodenal obstruction = polyhydramnious


pregnant patient at 37 weeks gestation, known case of RH immunization, fetus found to have anemia,
what is the management?
Delivery of the baby
Give blood to the fetus ( not written like this)

A

Pregnant in active labor (I think) She has history of recurrent vaginal HSV infections, next step?
A. Instrumental delivery
B. Speculum exam (not sure)/or vaginal exam
C. CS
D. Acyclovir

B To check for any active lesion in vagnia and cx. If positive then cs and acyclovir (‫ اﻟﺳﻛﺷن اھم ﺷﻲ ﻟﻠوﻻده‬


! Pregnant in 39 or 38 weeks came to hospital starting labora in fwe hours cervix 6 cm , spontaneous

rupture of membrane 20 hours ago ctg shows fetal bradycardia what is most appropriate
management?
— Stop oxytocin
— CS

B
! " Note to remember

- The patient had prolonged active phase of labor-> she was managed by amniotomy and oxytocin
- Now she is having arrested active phase-> which is managed by C/S!

Let’s Exclude!!
- Stop oxytocin-> I would do it as a next step (not the most appropriate) initiating in utero resuscitative measures while
I’m waiting for the C/S (Change of maternal position is a reasonable first treatment option, followed by O2, IV fluid, stop
oxytocin, administer toocolytic drugs.)
- Amnioinfusion -> is the second line option after in utero resuscitative measures (so, i will not choose it as a next step or
most appropriate)
- Ampicillin -> if he says next not the most appropriate


% Absolute contraindications to external cephalic version
Asymptomatic newly married female pt came for the clinic for general gyn evaluation What is the
highest diagnostic test?
A) General appearance
B) Digital pelvic exam ✅ ‫ اﻟﺻﺢ‬
C) Abdominal examination
D) history ✅ ✅ ‫ اﻷﺻﺢ‬

Pregnant & in labour was induced by oxytocin, CTG showing late deceleration with picture,
What to do to reverse condition ?
A- change mother position to sleep supine
B- give epidural anaesthesia
C- weird word CEASING oxytocin but i chose it

C

PROM ,, 34 weeks , What to give ?
A- Tocolytics
B- steroid
C- antibiotic
D- irrelevant

B

- female c/o came with missed abortion which drug you give her?
A-misoprostal '
B- oxytocin
there’s no D/C
‫ﺣﺳب أي اﺳﺑوع‬

‫وﺣدة ﻋﻧدھﺎ رﺑﺷر اﻛﺗوﺑك ﻓﻲ ال‬US ‫ ﻓﻲ ﺷوﯾﺔ ﻓﻠوﯾد ﻓﻲ اﻟﺑروﺗﻧﯾﺎم‬
‫ ﻛﻠﯾﻧﯾﻛﺎﻟﻲ ﺳ ﺗ ﯾ ﺑ ل‬
What eirlier indicator for ongoing blood loss ?
-tachycardia
-tachepnea
-hypotension
-decrease urine output

A

Young female presented to the ER complaining of general fatigue symptoms and dizziness and
mention that she is bleeding for 15 days since her period started. No mention of Examination or
current bleeding status. Lab: Hb 7 What is the most appropriate next step?
A- US
B- Start blood transfusion
C- Stop the bleeding and send her home

B

Female pregnant US showed Intrauterine fetal death what to do:
A- Inform patient
B- inform husband
C- Do other exams for fetal death

A


Pt with pcos and hairstim how to asses her hirsutism ?
A-testosterone level
B-gonadotrobin level

A

Qs immunoglobulin presented in breast milk?
A. IgG
B. IgM
C. IgE
D. IgA

D

Short senario about a women did IVF and got pregnant (positive pregnancy test), US should a mass 4
cm in rt tube, vitally stable. No mention for other things… she’s planned to do laparoscopy, what is
the management?
Hystroectomy
Bilateral tube removale
Rt salpingotomy (100% sure of this spelling)

C

Case about multi para, in active labor, cevix is 80% and dilated 7 cm and 0 station, she have mild
irregular contractions, CTG pic should 2 acceleration and HR 150 with mod varibality, what to do?
- augment the labor with oxytocin
- Reposition the mother and resuscitation
- Waiting

C

A 42w pregnant come to antenatal care. Head well fixed in pelvis and cervix is favorable. Most
appropriate management?
A. CTG
B. IOL
C. CS

B

! Pregnant 28 weeks came with bilateral breast mass it was movable and the size was3x4 cm, most
appropriate next step?
— reassurance and follow up after delivery
— bilateral breast ultrasound
— Bilateral breast mammogram
— MRI

B

Sign of Rh alloimmunization
A-Low middle cerebral artery peak systolic velocity
B-skin edema and ascites

B

Pregnant delivered her baby at home, the baby has onle bruses , what is the dx? Invst show : normal
plte and high pt and ptt
A-Thrombophilia
B-Itp
C-Factor X def
D-Hemorrhagic infants ( sometimes like this )

D

17 yrs Senario of primary dysmenorrhea severe pain affect on life and school attendance. She was on
NSAID and pain become less severe, she able to go school and do her life activities.. most appropriate
next step?
1-OCP
2-counseling and education of self care

2

Pregnant female 28GA with asymptomatic bacteruria tx:
A) oral ciprofloxacin
B) Oral nitrofurantoin
C) oral trimethoprim/sulfa

B

30 year old female pregnant with new symptoms of dyspnea, productive cough and fatigue. Doppler
US was negative what is the next test to do:
A: cta
B: d dimer
C: spirometery
D: ventilation and perfusion scan

D

Pregnant at 24 week all is good she’s just complaint of protruding mass from the vagina, diagnosed as
posterior vaginal wall Prolapse, what to do ?
A- reassure
B- Emergency delivery

A

Pregnant 9 weeks come with sever bleeding she says that there was gush of fluid and part on
examination os is open and tissue is seen (incomplete abortion)what’s most appropriate step?
A-Expectant
B-D&C

B (sever)

Patient with greysh vaginal discharge (odorless) and spores on microscope?
candida


Case of HELLP Syndrome, patient was refered from another hospital and came with headache. BP:
160/.. , high AST and ALT. What is the appropriate management?
- Immediate C-section.
- Induction of labor as delivery is anticipated.

B

Rational behind giving steroid before preterm labor?
Respiratory distress syndrome


What you will give to decrease risk of Respiratory distress syndrome? Glucocorticoids

A pregnant woman GA 30 weeks with preterm labour was given a Tocolytic. What is the rationale
behind giving tocolytics ?
A- To Delay the delivery until 37 weeks
B- To maximize the effect of steroids
C. To prevent PROM

B

Young female patient complaining of dymenorrhea and she have infertility since (i cant remeber the
duration but not more than 2 year) , what is the cause of her dysmenorrhea ?
A- fibroid
B- Endometriosis
C- adenomyosis

B

pregnant in 2nd trimester ( I think) with RUQ pain what is the cause?
Cholelithiasis
cholecystitis
liver capsules injury

B

female pregnant with hypertension and proteinuria, she has right upper quadrant pain what is the
reason ?
A- Distended Hepatic Capsule
B- Hepatic Rupture
C- Gallbladder Stone

A

! Gold standard to know GA (the pt amenorhea from 6-8 weeks, before 2 weeks do home pregnancy:

— US
— Quantitative bhog
— Start from pregnancy test

A

! Female patient LMP 6-8 weeks she can’t remember and did pregnancy test in home 2 weeks ago

and was positive she did it before and was negative, most appropriate next inx?
— quantitative Bhcg
— ultrasound
— ultrasound after 3-4 weeks

A

Primigravida, preterm known case of DM 1 came with sever contractions and closed cervix What to
give?
A. Steroids + insulin
B. Steroids + insulin +tocolytics.
C. Steroids and tocolytics
D. Insulin and tocolytics

A (Closed cervix)


T1DM 32 weeks came with cervical dilation no gush of fluid or blood is seen, cervix is 2cm she has
regular contractions and + fibronictin.
Insulin and steroids
Tocolytic and steroids
Tocolytic and insulin
Tocolytic insulin steroid
D (Dilated cervix)
Patient with hypertension, heavy bleeding and anemia, what contraceptive she can use?
A. Tube ligation
B. IUCD (If they mentioned hormonal IUCD OR levonorgestrel IUCD, this is the correct answer)
C. Condom (if They didn’t specify the type of IUCD in B)
D. OCP

B

Qs patient has irregular heavy menstrual bleeding ( i think they mentioned she has fibroid although
I’m not sure) , she doesn’t want to conceive for 2 years , what is the most appropriate contraceptive
method to give?
A-depoproveral injection
B-IUD
C-COC
D-progrstin only pils

Ans. IUD mirena? Or A

Pregnant lady, what marker you will find in urine to diagnose GDM:
-ketones
-Glucose

B

19 yrs primigravida 32 GA referred by family doctor having blood pressur 150/90 and proteinuria what
you will do?
A-Admit & measure blood pressure and look for proteinuria something like that
B-Refer her back for her family physician and daily monitor BP and protein in urine

A

Patient with long term history of DM type 1 in 12 weeks of gestation. HbA1C 12. Which of the
following complication is most likely to happen?
A. Preeclampsia
B. Polyhydroamnios
C. Oligohydromnios
D. IUGR

A

307- What is the rational of antihypertensive medicatio in preeclampsia?
A.decrease UGR
B. Decrease mothers’ mortality
c- Decrease fetus mortality

B



624- MRI of large fibroid and mensterural bleeding hg 7 what’s next step in management?
A correct anemia
B. Ocp
C. Myomectomy

A

Pregnant visits antenatal care clinic due to vaginal bleeding . What's your management?
Expectant ttt

You might also like